[obm-l] Re: [obm-l] cadeira de 3 pés

2023-01-23 Por tôpico Eduardo Wagner
O banco de 3 pernas não balança porque nosso mundo é tridimensional.
Não tem nada a ver com plano ou triângulo. Um banco de 3 pernas não balança
se for colocado
no teto de um carro.
No mundo 2D um banco de 2 pernas não balança, mas um de 3 pernas pode
balançar.
Em um mundo 4D uma cadeira de 4 pernas não balança.
Wbs
Wagner


Em dom., 22 de jan. de 2023 às 23:24, Claudio Buffara <
claudio.buff...@gmail.com> escreveu:

> Achei na internet duas explicações distintas para a estabilidade de uma
> cadeira (ou mesa ou banco) de 3 pés.
> Aqui estão:
> https://www.somatematica.com.br/curiosidades/c98.php
>
> http://colegiofarroupilha.com.br/site/qual-cadeira-e-mais-firme-a-que-tem-tres-ou-quatro-pes/
>
> Qual das duas é a explicação correta?
> Ou nenhuma das duas? E, nesse caso, qual a explicação?
>
> []s,
> Claudio.
>
> --
> Esta mensagem foi verificada pelo sistema de antivírus e
> acredita-se estar livre de perigo.

-- 
Esta mensagem foi verificada pelo sistema de antiv�rus e
 acredita-se estar livre de perigo.



Re: [obm-l] Algo errado com o enunciado?

2019-03-18 Por tôpico Eduardo Wagner
Prezado Pedro:

Relaxe. Não há nenhum conjunto obrigatório para os naturais. Cada um adota
o que quiser, com o zero ou sem o zero.
Em sequências costuma-se adotar o conjunto dos naturais sem o zero, pois
quando estamos contando elementos
de algum conjunto, a maioria das pessoas normais não começa a contar pelo
zero.
Em aritmética o zero deve ser incluído como elemento neutro da adição.
Oberve que nos axiomas de Peano, o zero não está incluído. Isso é
conveniente pois o axioma 4 é a base para o
Princípio da indução.
Enfim, essas questões de nomenclatura não são importantes.
Adote a que for mais conveniente ao objeto que você está estudando.
Att,
E, Wagner.

Em sáb, 16 de mar de 2019 às 14:41, Pedro José 
escreveu:

> Boa tarde!
>
> Já questionei uma vez aqui no sítio sobre um fato, para mim curioso.
> Estudara no ginásio e também no científico que os inteiros positivos,
> representado por um Z estilizado e um sinal de adição eram elementos do
> conjunto {0, 1, 2, 3,...} e os inteiros estritamente positivos teriam a
> representação por um Z estilizado um sinal de adição e um asterisco e
> seriam elementos de {1, 2, 3, 4, 5,...} Então havia interseção entre os
> conjuntos dos inteiros positivos e negativos que seria obviamente o {0}. O
> mesmo acontecia com os reais positivos.
> Ainda relembro Miguel Jorge e Dona Frida chamando a atenção entre positivo
> e estritamente positivo.
> Futuramente deparei-me com esse novo conceito.
> Outra coisa Miguel Jorge costumava começar seus livros com o capítulo 0
> para frisar que ele considerava zero natural, hoje não encontro uma posição
> pacífica, já vi livros onde o zero não é considerado natural. Todavia,
> nunca mais vi quem considere zero um interior positivo.
> Gostaria de saber a razão da mudança. Se a corrente que estudei era uma
> dissidência que não pegou ou se de fato ocorreu alguma mudança para nos
> adequarmos a um entendimento mais global??
>
> Saudações,
> PJMS
>
>
>
>
>
>
>
> Em qui, 7 de mar de 2019 às 06:10, Anderson Torres <
> torres.anderson...@gmail.com> escreveu:
>
>> Em qua, 6 de mar de 2019 às 16:41, marcone augusto araújo borges
>>  escreveu:
>> >
>> > Seja f uma função definida para todo inteiro positivo tal que
>> >
>> > i) f(0) = 1
>> > ii) f(2n + 1) =  2f(n) + 1
>> > iii) f(2n) = 3f(n)
>> > .
>> > .
>> > .
>> >
>> > se vale para todo inteiro POSITIVO, porque começa com f(0)?
>>
>> Qual é a origem do problema?
>> Talvez tenha sido um mero ato-falho do examinador. Afinal, não me
>> parece que o problema prossegue insolúvel se supusermos "naturais" em
>> vez de "inteiros positivos".
>>
>> >
>> > --
>> > Esta mensagem foi verificada pelo sistema de antivírus e
>> > acredita-se estar livre de perigo.
>>
>> --
>> Esta mensagem foi verificada pelo sistema de antivírus e
>>  acredita-se estar livre de perigo.
>>
>>
>> =
>> Instru�ões para entrar na lista, sair da lista e usar a lista em
>> http://www.mat.puc-rio.br/~obmlistas/obm-l.html
>> =
>>
>
> --
> Esta mensagem foi verificada pelo sistema de antivírus e
> acredita-se estar livre de perigo.

-- 
Esta mensagem foi verificada pelo sistema de antiv�rus e
 acredita-se estar livre de perigo.



[obm-l] Re: [obm-l] Re: [obm-l] TRIÂNGULO

2019-03-11 Por tôpico Eduardo Wagner
Analítica. Adote AE como unidade de comprimento.
Resp: PQ/QR = 7/5

Em sáb, 9 de mar de 2019 às 12:40, Anderson Torres <
torres.anderson...@gmail.com> escreveu:

>
>
>
> Em qui, 7 de mar de 2019 às 07:47, Vanderlei Nemitz 
> escreveu:
>
>> Só enxerguei uma saída usando geometria analítica. Alguma ideia?
>> Muito obrigado!
>>
>> *Dado um triângulo ABC, com Â= 90º, D é o ponto médio de BC, F é o ponto
>> médio de AB, E é o ponto médio de AF e G o ponto médio de FB. AD intersecta
>> CE, CF, CG em P, Q e R respectivamente. Determine a razão PQ/QR.*
>>
>>
> A ideia que pensei foi usar Razão Cruzada.
>
> https://www.cut-the-knot.org/pythagoras/Cross-Ratio.shtml
> Mas só isso não vai adiantar.
>
>
>
>>
>>
>>
>>
>> 
>>  Livre
>> de vírus. www.avast.com
>> .
>>
>> <#m_5191055488509645045_m_3774298393707173559_m_6555290746475537769_DAB4FAD8-2DD7-40BB-A1B8-4E2AA1F9FDF2>
>>
>> --
>> Esta mensagem foi verificada pelo sistema de antivírus e
>> acredita-se estar livre de perigo.
>
>
> --
> Esta mensagem foi verificada pelo sistema de antivírus e
> acredita-se estar livre de perigo.

-- 
Esta mensagem foi verificada pelo sistema de antiv�rus e
 acredita-se estar livre de perigo.



[obm-l] Re: [obm-l] Dúvida conceitual (equações)

2018-10-16 Por tôpico Eduardo Wagner
Considere multiplicidades.

Em dom, 14 de out de 2018 às 06:38, Vanderlei Nemitz 
escreveu:

> Bom dia!
> Na seguinte questão, que me foi apresentada por um aluno, a resposta
> proposta é a alternativa C (1/2). Eu sempre pensei que apenas
> considerávamos multiplicidades em equações polinomiais. Como essa é uma
> equação exponencial, obtive a resposta B (-1/2). O que é correto pensar?
>
> O produto das raízes da equação 16.4^3x - 40.4^2x + 17.4^x - 2 = 0 é igual
> a:
> A) 1
> B) - 0,5
> C) 0,5
> D) - 1
> E) 0
>
> Muito obrigado!
>
> --
> Esta mensagem foi verificada pelo sistema de antivírus e
> acredita-se estar livre de perigo.

-- 
Esta mensagem foi verificada pelo sistema de antiv�rus e
 acredita-se estar livre de perigo.



Re: [obm-l] Re: [obm-l] Questão do IME

2018-07-14 Por tôpico wagner

Brilhante!


Quoting Claudio Buffara :


Os prolongamentos de DM e EN se intersectam num mesmo ponto P pertencente a
AB.
Pra ver isso, repare que os triângulos DCM e PAM são semelhantes (razão de
semelhança = 2).
Idem para os triângulos EFN e PNB.
Como, no triângulo PDE (que é isósceles), vale PM/PD = PN/PE = 1/3,
concluímos que MN é paralelo a DE.

[]s,
Claudio.


2018-07-13 12:13 GMT-03:00 Vanderlei Nemitz :


Sejam dois quadrados ABCD e ABEF, tendo um lado comum AB, mas não situados
num mesmo plano. Sejam M e N pertencentes, respectivamente, às diagonais AC
e BF tais que AM/AC = BN/BF = 1/3. Mostre que MN é paralelo a DE.

Alguém poderia ajudar?
Obrigado,
Vanderlei

--
Esta mensagem foi verificada pelo sistema de antivírus e
acredita-se estar livre de perigo.


--
Esta mensagem foi verificada pelo sistema de antiv?rus e
 acredita-se estar livre de perigo.





--
Esta mensagem foi verificada pelo sistema de antiv�rus e
acredita-se estar livre de perigo.

=
Instru��es para entrar na lista, sair da lista e usar a lista em
http://www.mat.puc-rio.br/~obmlistas/obm-l.html
=


Re: [obm-l] Re: [obm-l] Re: [obm-l] Re: [obm-l] União de Dois Conjuntos

2018-07-04 Por tôpico wagner

Não resisto:

A futura mãe, grávida, após os exames, pergunta ao médico:
"É menino ou menina?"
Resposta do médico; SIM.



Quoting Claudio Buffara :


A união de dois conjuntos é definida com base no conectivo lógico "OU" (x
pertence a A união B <==> x pertence a A  OU  x pertence a B).
E, em matemática (e em lógica), o "OU" não é exclusivo (ao contrário do uso
quotidiano deste conectivo).
Ou seja, dadas as proposições P e Q, a proposição composta "P OU Q" será
verdadeira em três situações:
P verdadeira e Q falsa,
P falsa e Q verdadeira, e
P e Q ambas verdadeiras.
Assim, o matemático é o sujeito que, quando perguntado se prefere açúcar ou
adoçante no seu café, responde "Sim".

[]s,
Claudio.


2018-07-04 17:56 GMT-03:00 Ronei Lima Badaró :


Acredito que seja para ser didático já que o "ou" em casos do cotidiano
pode ser excludente.

Em Qua, 4 de jul de 2018 17:52, Alexandre Antunes <
prof.alexandreantu...@gmail.com> escreveu:



Nessa definição ele separa apenas na parte de A (sem a parte comum a B),
parte de B (sem a parte comum a A) e a interseção  entre A e B.

Em Qua, 4 de jul de 2018 17:14, Luiz Antonio Rodrigues <
rodrigue...@gmail.com> escreveu:


Olá, boa tarde!
Eu achei a definição abaixo na Wikipedia.
Não entendi porque tenho que considerar a intersecção também...
Alguém pode me ajudar?
Muito obrigado!
Luiz

The *union of two sets* A and B is the *set* of elements which are in
A, in B, or in *both* A and B. For example, if A = {1, 3, 5, 7} and B =
{1, 2, 4, 6} then A ∪ B = {1, 2, 3, 4, 5, 6, 7}.

--
Esta mensagem foi verificada pelo sistema de antivírus e
acredita-se estar livre de perigo.



--
Esta mensagem foi verificada pelo sistema de antivírus e
acredita-se estar livre de perigo.



--
Esta mensagem foi verificada pelo sistema de antivírus e
acredita-se estar livre de perigo.



--
Esta mensagem foi verificada pelo sistema de antiv?rus e
 acredita-se estar livre de perigo.





--
Esta mensagem foi verificada pelo sistema de antiv�rus e
acredita-se estar livre de perigo.

=
Instru��es para entrar na lista, sair da lista e usar a lista em
http://www.mat.puc-rio.br/~obmlistas/obm-l.html
=


Re: [obm-l] boatos sobre elon lages lima

2017-05-24 Por tôpico wagner

Lamentavelmente é verdade.


Quoting Israel Meireles Chrisostomo :


É verdade que o Elon morreu?Fiquei chocado com essa notícia, o pessoal aqui
poderia confirmar a veracidade dessa notícia?

--
Esta mensagem foi verificada pelo sistema de antiv?rus e
 acredita-se estar livre de perigo.





--
Esta mensagem foi verificada pelo sistema de antiv�rus e
acredita-se estar livre de perigo.

=
Instru��es para entrar na lista, sair da lista e usar a lista em
http://www.mat.puc-rio.br/~obmlistas/obm-l.html
=


Re: [obm-l] Re: [obm-l] Geometria

2017-04-19 Por tôpico wagner

Brilliant!


Quoting Julio César Saldaña :


Imagino que D esteja sobre BC. Se for esse o caso:

ABD e AEC são congruentes.

Ángulo BAD = ángulo ECA e por isso ângulo DFC = 60, logo BEFD é inscritível.

EB = 2. BD e como ângulo B = 60 então ângulo EDB=90.

Como BEFD é inscritível então ângulo BFE=90 e finalmente ângulo BFC=90

Julio Saldaña


-- Mensaje original ---
De : obm-l@mat.puc-rio.br
Para : obm-l@mat.puc-rio.br
Fecha : Mon, 17 Apr 2017 11:55:34 -0300
Asunto : [obm-l] Geometria

Bom dia a todos,

Gostaria de uma ajuda com o seguinte problema:

Dado um triângulo equilátero ABC, tal que sobre o lado AB  
tenhamos um ponto

E e sobre o AC tenhamos um ponto D, com AE=BD=AB/3. Se as cevianas AD e CE
intersectam  no ponto F, qual a medida do ângulo BFC?

Grato pela atenção.

Abraços,

Marcelo




"Matemática é o alfabeto com o qual Deus escreveu o Universo"
Galileu Galilei

--
Esta mensagem foi verificada pelo sistema de antivírus e
acredita-se estar livre de perigo.


__
Si desea recibir, semanalmente, el Boletín Electrónico de la PUCP, ingrese a:
http://www.pucp.edu.pe/puntoedu/suscribete/


--
Esta mensagem foi verificada pelo sistema de antivírus e
acredita-se estar livre de perigo.

=
Instruções para entrar na lista, sair da lista e usar a lista em
http://www.mat.puc-rio.br/~obmlistas/obm-l.html
=





--
Esta mensagem foi verificada pelo sistema de antiv�rus e
acredita-se estar livre de perigo.

=
Instru��es para entrar na lista, sair da lista e usar a lista em
http://www.mat.puc-rio.br/~obmlistas/obm-l.html
=


Re: [obm-l] Re: [obm-l] Re: [obm-l] Re: [obm-l] Re: [obm-l] PDF sobre OLimpíadas

2016-06-28 Por tôpico wagner

Caro Israel:
Excelente. Parabéns pelo incrível trabalho.
Os jovens estudantes olímpicos vão adorar.
Abraço,
E. Wagner.


Quoting Sergio Lima <sergi...@smt.ufrj.br>:


Caro Israel,

Dizer que o trabalho é ótimo, incrível e espetacular é até pouco.
O trabalho é surreal. O esforço deve ter sido descomunal, certamente
fruto de anos de trabalho, digitação, diagramação, preparação das figuras
etc.

Guardei para saborear com calma ao londo dos próximos dias, meses e anos.

Abraço,
sergio




2016-06-24 21:47 GMT-03:00 Israel Meireles Chrisostomo <
israelmchrisost...@gmail.com>:


Obrigado gente!

Em 24 de junho de 2016 20:32, Carlos Gomes <cgomes...@gmail.com> escreveu:


Acabei de folhear rapidamente, mas mesmo rápido já dá para perceber a
qualidade do seu trabalho Israel. Parabéns, a comunidade olímpica agradece!

Abraço, Cgomes.

Em 24 de junho de 2016 20:13, Mauricio de Araujo <
mauricio.de.ara...@gmail.com> escreveu:


Israel, muito bom este trabalho!! vou dar uma olhada e, se for o caso,
sugerirei alguma adequação... parabéns!!

Em 24 de junho de 2016 19:25, Israel Meireles Chrisostomo <
israelmchrisost...@gmail.com> escreveu:


Olá pessoal estou compartilhado um PDF que escrevi, acrescentei vários
problemas:

http://media.wix.com/ugd/3eea37_3049c428c55948f2b8bb069834275f50.pdf

Quem tiver alguma sugestão ou correção, por favor envie para o meu
email, pois muitas pessoas podem se beneficiar com o acerto ou mesmo se
prejudicar com o erro.

Obrigado.

israelmchrisost...@gmail.com

--
Esta mensagem foi verificada pelo sistema de antivírus e
acredita-se estar livre de perigo.





--

Abraços,
oɾnɐɹɐ ǝp oıɔıɹnɐɯ


--
Esta mensagem foi verificada pelo sistema de antivírus e
acredita-se estar livre de perigo.




--
Esta mensagem foi verificada pelo sistema de antivírus e
acredita-se estar livre de perigo.




--
Esta mensagem foi verificada pelo sistema de antivírus e
acredita-se estar livre de perigo.


--
Esta mensagem foi verificada pelo sistema de antiv�rus e
 acredita-se estar livre de perigo.






This message was sent using IMP, the Internet Messaging Program.



--
Esta mensagem foi verificada pelo sistema de antiv�rus e
acredita-se estar livre de perigo.


=
Instru��es para entrar na lista, sair da lista e usar a lista em
http://www.mat.puc-rio.br/~obmlistas/obm-l.html
=


Re: [obm-l] Trigonometria

2015-12-11 Por tôpico wagner

Obrigado, Israel. Gostei muito.
Abraço,
E. Wagner.


Quoting Israel Meireles Chrisostomo <israelmchrisost...@gmail.com>:


Olá amigos da obm, estou passando para divulgar um texto que, após muitas
modificações, seja acrescentando novos problemas ou outras soluções,
acredito estar terminado(mesmo assim se virem erros por favor me digam :) ,
ficarei feliz se o texto estiver integralmente correto em termos
matemáticos, eis aqui o link para o texto:

http://media.wix.com/ugd/3eea37_21c6dd3b7aeb44279a7db6f205ba1686.pdf

--
Esta mensagem foi verificada pelo sistema de antiv?rus e
 acredita-se estar livre de perigo.







This message was sent using IMP, the Internet Messaging Program.



--
Esta mensagem foi verificada pelo sistema de antivírus e
acredita-se estar livre de perigo.


=
Instruções para entrar na lista, sair da lista e usar a lista em
http://www.mat.puc-rio.br/~obmlistas/obm-l.html
=


[obm-l] Ajuda

2014-12-07 Por tôpico Wagner
Olá à todos
Por favor :

Se um cubo tem 3 dimensões , e um hipercubo pode ter  4, 5 , 6 ..
Pergunto :
Em uma esfera , quantas dimensões tenho ?
E se for inscrita , ou circunscrita duas esferas , quantas dimensões terei ?
Grato
Wagner
-- 
Esta mensagem foi verificada pelo sistema de antivírus e
 acredita-se estar livre de perigo.



Re: [obm-l] Integral

2013-10-24 Por tôpico wagner


f(x)=ln(sqrt(x)-1)-ln(sqrt(x)+1)



Quoting saulo nilson saulo.nil...@gmail.com:


x=tany

R=lnseny=lnx/(1+x^2)



2013/10/23 Prof Marcus marcusaureli...@globo.com


Alguém pode dar uma ideia nessa integral?

integral dx/x^1/2(x+1)

obrigado



--
Esta mensagem foi verificada pelo sistema de antivírus e
acredita-se estar livre de perigo.


--
Esta mensagem foi verificada pelo sistema de antivírus e
 acredita-se estar livre de perigo.







This message was sent using IMP, the Internet Messaging Program.



--
Esta mensagem foi verificada pelo sistema de antivírus e
acredita-se estar livre de perigo.


=
Instruções para entrar na lista, sair da lista e usar a lista em
http://www.mat.puc-rio.br/~obmlistas/obm-l.html
=


[obm-l] Re: [obm-l] aumento hipotenúzico

2013-07-10 Por tôpico Wagner
426% ???

  - Original Message - 
  From: Thelio Gama 
  To: obm-l@mat.puc-rio.br 
  Sent: Wednesday, July 10, 2013 12:10 PM
  Subject: [obm-l] aumento hipotenúzico


  Prezados mestres,


  poderiam dar uma ajuda na seguinte questão?



  Os catetos de um triângulo retângulo medem 4cm e 3cm. Se a medida do primeiro 
cateto for reduzida em 10% e a do segundo for aumentada em 60%, qual será a 
taxa de aumento da hipotenusa?



  Obrigado a todos,


  abraços!

  -- 
  Esta mensagem foi verificada pelo sistema de antivírus e 
  acredita-se estar livre de perigo. 
-- 
Esta mensagem foi verificada pelo sistema de antivírus e
 acredita-se estar livre de perigo.



Re: [obm-l] Problema de Geometria

2013-04-28 Por tôpico Wagner
Olá senhores
Desculpe a minha pergunta
Há tempos sou assinante desta, e por vezes vejo termos, que para mim são 
desconhecidos, como por exemplo o teorema abaixo citado.
Sou graduando em matemática 4º semestre  ( licenciatura )
Daí pergunto:
Haveria algum site , ou coleção de livros , ou DVD's , que abranje  todos os 
tópicos de matemática 
Grato
Wagner
PY2RPD
  - Original Message - 
  From: Rogerio Ponce 
  To: obm-l@mat.puc-rio.br 
  Sent: Sunday, April 28, 2013 9:17 PM
  Subject: Re: [obm-l] Problema de Geometria


  Ola' Raphael,

  e' so' aplicar o Teorema de Menlaus ao triangulo AOD com a reta CB, obtendo:

  AC * FD * OB = DC * OF * AB
  ou seja

  FD = 2 * OF

  Como EF = OE - OF
  entao EF = (a/2) - (b/3)


  []'s

  Rogerio Ponce




  2013/4/28 Raphael Feijao raphaelfei...@hotmail.com

O segmento AB é o diametro de uma circunferencia de centro O. Toma-se um 
ponto C desse círculo e prolonga-se o segmento AC de um segmento CD igual a AC. 
O segmento OD corta a circunferencia em E e corta o segmento BC em F. Se AB=a e 
OD=b. Calcule EF. 




__ Informação do ESET Smart Security, versão da vacina 8276 (20130428) 
__

A mensagem foi verificada pelo  ESET Smart Security.

part000.txt - esta OK
part001.htm - esta OK

http://www.eset.com



Re: [obm-l] Ajuda

2012-11-19 Por tôpico Wagner
Olá prezado Sr vanderlei
Grato pela ajuda
Muito bom o algoritmo, mas haveria uma fórmula generalizada para tal resolução.
Penso na seguinte possibilidade X+Y+Z= 1000,  por exemplo, então fica   meio 
dificil  um simples arranjo
Gratíssimo a todos
Wagner
  - Original Message - 
  From: Vanderlei * 
  To: obm-l@mat.puc-rio.br 
  Sent: Monday, November 19, 2012 11:21 AM
  Subject: Re: [obm-l] Ajuda


  Você quer saber como chegamos nas 36 soluções? Existem vários caminhos, mas 
um deles é:

  Representando por o uma unidade, duas possíveis soluções são: oo+o+ e 
o++oo, que nada mais são do que as representações de 2+1+4 e 5+0+2. Assim, 
toda solução é uma permutação de 9 símbolos, 7 dos quais são o e 2 são +, 
ou seja, o número de soluções inteiras e não negativas é dado pelo número de 
permutações de 9 elementos, com 7 repetições de o e 2 repetições de +.

  N = 9!/(7!.2!) = (9.8)/2 = 36

  Vanderlei


   
  Em 19 de novembro de 2012 03:44, Wagner w...@bol.com.br escreveu:

Olá 
Na equação diofantina x+y+z=7 
Usando análise combinatória , existe 36 possiveis soluções
Ma eu não consegui resolver
Motivo pelo qual estou pedindo esta ajuda aos senhores
Grato
Wagner



[obm-l] Ajuda

2012-11-18 Por tôpico Wagner
Olá 
Na equação diofantina x+y+z=7
Usando análise combinatória , existe 36 possiveis soluções
Ma eu não consegui resolver
Motivo pelo qual estou pedindo esta ajuda aos senhores
Grato
Wagner

Re: [obm-l] Re: [obm-l] Valor máximo para a soma dos senos

2011-11-24 Por tôpico wagner

Considere que a soma dos senos é p/R.
Fixe uma circunferência e considere todos os triângulos inscritos.
A soma dos senos será máxima quando o perímetro for máximo.
Ok.
Fixe um lado do triângulo e varie sobre a circunferência o vértice oposto.
O perímetro do triângulo será máximo quando os dois lados que estavam  
variando forem iguais. Repetindo o mesmo para os outros lados, o  
máximo do perímetro ocorre quando o triângulo inscrito for equilátero.  
A soma máxima dos senos é

3.sqrt(3)/2.



Quoting Ralph Teixeira ralp...@gmail.com:


Procure derivadas parciais. :)

2011/11/24 João Maldonado joao_maldona...@hotmail.com




Recentemente vi um problema na lista sobre como calcular a soma  dos 3
senos de um triângulo,  em que a resposta foi p/R
Fiquei pensando então qual deveria ser o valor máximo para esta soma

Fiz  assim:

Dada uma circunferência de raio R,   e um dos  lados do triângulo, que
chamaremos de w,  temos  necessariamente que um  ângulo (que chamaremos de
W) já está determinado,  o valor máximo de  p então seria o  valor máximo
da  soma dos outros lados.

Como a² + b² -  2abcosW = w² -  (a+b)² - 2ab(1+co sW) = w²

Como w.h/2 = a.b.senW/2,  temos  que ab = h.w/senW
Logo  (a+b)² = w² + 2hw(1+cosW)/senW

Como   o lado e o ângulo já estão determinados,  e 1+cosW0  e senW0,   o
valor máximo de a+b se dá no valor máximo da altura,  vem que  o triângulo
é isósceles

E  fazendo  y = 90-w/2

Daí a soma dos 3 senos  é 2sen(y) + sen(2y)  que derivando dá 2(cos(y) +
cos(2y)  e igualando a 0

2cos²(y) +  cos(y) - 1 = 0 -  cos(y) = 1/2 ou  -1

Substituindo  temos  que o valor máximo é cos(y) = 1/2  e a soma vale
3(3)^(1/2)/2




Queria saber se há alguma derivada  de 2 variáveis,  no caso  a e b que
desse o valor  máximo se sen(a) +  sen(b) + sen(a+b)
Já  ouvi falar em integral dupla (na verdade só  ouvi falar, não faço a
mínima idéia do que seja,  mas pensei  que tivesse alguma coisa a ver com a
 integral de 2 variáveis)
Entretanto procurei derivada dupla no google e não encontrei nada

Existe isso?

[]'s

João









This message was sent using IMP, the Internet Messaging Program.



=
Instruções para entrar na lista, sair da lista e usar a lista em
http://www.mat.puc-rio.br/~obmlistas/obm-l.html
=


Re: [obm-l] Outro problema do Morgado II

2011-11-22 Por tôpico wagner

Atenção:
Azincourt se enganou. A resposta envolve o raio R da circunferência  
circunscrita e não o raio r da inscrita.
Entretanto, ele quase chegou na resposta. Sigam a partir do resultado  
que ele obteve:

4(p^2)r/abc = 4ppr/abc = 4pS/4RS = p/R (esta é a resposta).
Abs.
W.



Quoting Azincourt Azincourt aazinco...@yahoo.com.br:


Boa noite!

Obrigado pela ajuda com os problemas anteriores! Tenho um outro para  
 ser debatido.



Problema 175
Determine a soma dos senos dos ângulos de um triângulo de   
semiperímetro p e raio da circunferência inscrita r.


Eu tentei o seguinte raciocínio, que acabou por não me levar muito longe:


S = (1/2)ab sen C = (1/2) bc sen A = (1/2) ac sen B

Isolei os senos em cada uma das equacoes anteriores, de forma que   
sen A + sen B + sen C = (2S/ab) + (2S/bc) + (2S/ac)

Mas (2S/ab) + (2S/bc) + (2S/ac) = 2S (a+b+c)/abc

Usando S = pr, e a + b + c = 2p, obtive

sen A + sen B + sen C = 2pr*2p/abc = 4(p^2)r/abc

Porém, a partir daí, não consegui um modo prático de substituir o   
produto abc por uma expressão envolvendo p e r apenas.


Alguém pode me ajudar?

Muito obrigado!






This message was sent using IMP, the Internet Messaging Program.



=
Instruções para entrar na lista, sair da lista e usar a lista em
http://www.mat.puc-rio.br/~obmlistas/obm-l.html
=


Re: [obm-l] OBM terceira faze nivel 3 - Gabarito duvidoso

2011-01-26 Por tôpico wagner

Oi gente:

Não há solução oficial da OBM deste problema porque a solução oficial
eu fiquei de fazer, mas infelizmente, esqueci de enviar. Peço desculpas.
Estamos na semana Olímpica e prometo que, na semana que vem,  
publicarei uma solução bem legal.


Abraços,

E. Wagner.


Quoting Ralph Teixeira ralp...@gmail.com:


Concordo contigo, João: projetando um cubo ABCD-EFGH num plano paralelo a
BDE, temos um hexágono regular de área raiz(3) -- então aquela solução que
dizia raiz(6)-1 não pode estar certa.

Isto dito, há algum motivo para acreditar que a solução oficial da OBM é a
do link dado?

Abraço,
Ralph

2011/1/26 João Maldonado joao_maldona...@hotmail.com


OBM 2010 Terceira Fase


PROBLEMA 3
Qual é a maior sombra que um cubo sólido de aresta 1 pode ter, no sol a
pino?
Observação: Entende-se “maior sombra de uma figura no sol a pino” como a
maior área possível para a
projeção ortogonal da figura sobre um plano.

O que me perturba é que a resolução desse site dá que a maior sombra  tem
área sqrt(6) - 1

http://files.supergel57.webnode.com.br/20496-53215537a7/Resolu%C3%A7%C3%A3o%208.pdf

Mas tome o seguinte:
Coloque qualquer vértice do cubo (A) em contato com um plano de modo que o
vértice oposto (B) forme uma reta perpendicular ao plano.
As 3 arestas que saem desse vértice formam o mesmo ângulo com o plano. As 3
faces que saem desse vértice formam o mesmo ângulo com o plano.
Logo os vértices adjacentes formam um tetraedro com base regular e sua
lateral composta por triêngulos retângulos.
Os vértices não adjacentes (com exceção  de B) formam um tetraedro com base
regular e sua lateral composta por triângulos equiláteros.
Considerando a reta AB, esta é altura dos tetraedros. Logo fica fácil
calcular a distância de AB e os vértices (2/3 da altura do triângulo da
base).
Essa distância é sqrt(6)/3 para todo os 6 vétices (não contando com A e B),
já que os dois tetraedros tem a mesma base.
Ou seja, é formado um hexágono de raio sqrt(6)/3 cuja área mede
6.(sqrt(6)/3)² sqrt(3)/4 = sqrt(3)  sqrt(6) - 1.

Pergunta:
Qual das duas soluções está errada?








This message was sent using IMP, the Internet Messaging Program.



=
Instru��es para entrar na lista, sair da lista e usar a lista em
http://www.mat.puc-rio.br/~obmlistas/obm-l.html
=


Re: [obm-l] OBM terceira faze nivel 3 - Gabarito duvidoso

2011-01-26 Por tôpico wagner

Oi gente:

Não há solução oficial da OBM deste problema porque a solução oficial
eu fiquei de fazer, mas infelizmente, esqueci de enviar. Peço desculpas.
Estamos na semana Olímpica e prometo que, na semana que vem,  
publicarei uma solução bem

legal.

Abraços,

E. Wagner.





Quoting Rogerio Ponce abrlw...@gmail.com:


Ola' Joao,
conforme eu ja' havia dito, o hexagono em questao e' REGULAR.
E não tem nenhuma diagonal sqrt(3) paralela ao plano horizontal.
Voce e a OBM estao errando nisso.
Se voce mesmo nao chegar 'a uma solucao bonitinha, mais tarde eu explico
melhor...
[]'s
Rogerio Ponce


2011/1/26 João Maldonado joao_maldona...@hotmail.com


 Boa Tarde Rogério,

Sua resolução está perfeitamente correta para um hexágono REGULAR , como
voce disse . Acontece que para o hexágono ser regular todas as diagonais
sqrt(3) do cubo têm que estar paralelas ao plano, e consequentemente entre
si mesmas. E isso não é possível já que as diagonais maiores do cubo não são
paralelas. Essa solução geraria um hexágono sim, mas não um hexágono
regular.  A medida de seus outros raios seria menor que sqrt(3)/2, e
consequentemente a área seria menor que 9*sqrt(3)/8.
Sua solução é justamente a do link   (veja como o hexágono não é regular),
e a área desse hexágono vale sqrt(6) - 1.
Agora veja o que estou dizendo, não estou falando que sqrt(3) é a MAIOR
área da projeção ortogonal do cubo, pode até ser, mas sim que é uma POSSÍVEL
área. Logo como sqrt(3)  sqrt(6) - 1, a solução da OBM estaria errada.

Ainda estou tentando achar um erro em minha solução mas ainda não
encontrei. É isso que queria saber, minha solução está errada o a solução
oficial que está errada?

Grato,
João
--
Date: Wed, 26 Jan 2011 10:56:27 -0200

Subject: Re: [obm-l] OBM terceira faze nivel 3 - Gabarito duvidoso
From: abrlw...@gmail.com

To: obm-l@mat.puc-rio.br

Ola' Joao,
eu diria que as duas solucoes estao erradas.
A maior projecao gera um hexagono REGULAR. Como a diagonal do cubo mede
sqrt(3), este sera' o diametro do circulo circunscrito ao hexagono. Logo a
area do hexagono deve ser
   6* [sqrt(3)/2 * sin60] * [sqrt(3)/2 * cos60]
Ou seja,
   9*sqrt(3)/8

[]'s
Rogerio Ponce



2011/1/26 João Maldonado joao_maldona...@hotmail.com

 OBM 2010 Terceira Fase


PROBLEMA 3
Qual é a maior sombra que um cubo sólido de aresta 1 pode ter, no sol a
pino?
Observação: Entende-se “maior sombra de uma figura no sol a pino” como a
maior área possível para a
projeção ortogonal da figura sobre um plano.

O que me perturba é que a resolução desse site dá que a maior sombra  tem
área sqrt(6) - 1

http://files.supergel57.webnode.com.br/20496-53215537a7/Resolu%C3%A7%C3%A3o%208.pdf

Mas tome o seguinte:
Coloque qualquer vértice do cubo (A) em contato com um plano de modo que o
vértice oposto (B) forme uma reta perpendicular ao plano.
As 3 arestas que saem desse vértice formam o mesmo ângulo com o plano. As 3
faces que saem desse vértice formam o mesmo ângulo com o plano.
Logo os vértices adjacentes formam um tetraedro com base regular e sua
lateral composta por triêngulos retângulos.
Os vértices não adjacentes (com exceção  de B) formam um tetraedro com base
regular e sua lateral composta por triângulos equiláteros.
Considerando a reta AB, esta é altura dos tetraedros. Logo fica fácil
calcular a distância de AB e os vértices (2/3 da altura do triângulo da
base).
Essa distância é sqrt(6)/3 para todo os 6 vétices (não contando com A e B),
já que os dois tetraedros tem a mesma base.
Ou seja, é formado um hexágono de raio sqrt(6)/3 cuja área mede
6.(sqrt(6)/3)² sqrt(3)/4 = sqrt(3)  sqrt(6) - 1.

Pergunta:
Qual das duas soluções está errada?










This message was sent using IMP, the Internet Messaging Program.



=
Instru��es para entrar na lista, sair da lista e usar a lista em
http://www.mat.puc-rio.br/~obmlistas/obm-l.html
=


[obm-l] Gabarito duvidoso

2011-01-26 Por tôpico wagner

Oi gente:

Não há solução oficial da OBM deste problema porque a solução oficial
eu fiquei de fazer, mas infelizmente, esqueci de enviar. Peço desculpas.
Estamos na semana Olímpica e prometo que, na semana que vem,  
publicarei uma solução bem

legal.

Abraços,

E. Wagner.



This message was sent using IMP, the Internet Messaging Program.



=
Instruções para entrar na lista, sair da lista e usar a lista em
http://www.mat.puc-rio.br/~obmlistas/obm-l.html
=


[obm-l] Integral

2009-10-07 Por tôpico Wagner
Olá a todos da lista 
Tenho uma questão:
Provar que a integral indefinida de 1 é X
Grato
Wagner


__ Informação do ESET NOD32 Antivirus, versão da vacina 4487 (20091007) 
__

A mensagem foi verificada pelo  ESET NOD32 Antivirus.

http://www.eset.com



[obm-l] Completar os quadrados

2009-08-05 Por tôpico Wagner
Olá colegas
Estou com a seguinte dúvida:
Na equação x^2-6x+1=(x-3)^2-9+1
Ok eu não entendi como foi feita esta passagem
Grato pela ajuda
Wagner

[obm-l] Dúvida

2009-08-05 Por tôpico Wagner
Olá a todos 
Colegas eu posso dizer que x^2 =SQR 2^2
Porque?
Grato
Wagner

[obm-l] ajuda

2009-08-02 Por tôpico Wagner
Olá a todos da lista !
Estou com a seguinte dúvida:
a = alfa
b = beta

1/a=1/b+1 daí a=b/b+1 , b=a/a-1
Eu não entendi como se chega a esses resultados.
Algum colega poderia me ajudar por favor ?
Grato
Wagner

[obm-l] Off-Topic Matlab

2009-07-24 Por tôpico Wagner
Olá a todos os colegas da lista !
Eu tenho a seguinte dúvida:
Se no MatLab eu tiver , a=2+2; e salvar a, quando eu dou o comando load a, 
só tenho o resultado da operação .
Como eu poderia salvar ,e carregar inclusive a equação , para mostrar tudo ?
Grato
Wagner

[obm-l] Re: [obm-com] Membro da Comissão Nacional de Olimpíadas de Matemática é premiado

2009-07-22 Por tôpico wagner

Parabéns, Gugu!

Quoting Olimpiada Brasileira de Matematica o...@impa.br:


Prezados professores:



É com grande satisfação que a Secretaria a OBM informa que o   
professor Carlos Gustavo Moreira ganhou o prêmio UMALCA-2009-
União Matemática para a América Latina e Caribe. Este prêmio   
contempla matemáticos de até 45 anos que se destacaram
na América Latina por sua contribuição a essa ciência. O prof.   
Carlos Gustavo é ex-olímpico, pesquisador do IMPA,
membro da Comissão nacional de Olimpíadas de Matemática, que   
organiza a Olimpíada Brasileira de Matemática e editor da revista   
Eureka!.
A Secretaria agradece a dedicação do prof. Carlos Gustavo e o   
parabeniza pelo prêmio.




Maiores informações:

http://www.impa.br/opencms/pt/destaques/memoria/2009/umalca_prize.html

--
Secretaria da Olimpíada Brasileira de Matemática Estrada Dona   
Castorina, 110 Jd. Botânico, Rio de Janeiro - RJ, 22460-320, Brasil

Tel: 55-21-25295077 Fax:55-21-25295023
e-mail: o...@impa.br web site: www.obm.org.br





This message was sent using IMP, the Internet Messaging Program.



=
Instruções para entrar na lista, sair da lista e usar a lista em
http://www.mat.puc-rio.br/~obmlistas/obm-l.html
=


[obm-l] Ajuda

2009-05-31 Por tôpico Wagner
Olá caros mestres da lista !
Estou precisando de uma ajuda !
No livro Manual prático de cálculo diferencial e integral na pag 129 temos:
Se lny = x lna , resolvamos com relação a x e teremos a função inversa x = 
(1/lna) *ln y
Eu não entendi essa passagem, porque 1/ln a ?
Poderia alguma mestre me esclarecer
MUito obrigado
Wagner
São Paulo/Capital

Re: [obm-l] Triangulo oco - centro de gravidade

2009-05-10 Por tôpico wagner

Caros amigos:

Ando sumido da lista. Mas leio frequentemente.
Acrescento um fato interessante sobre o centro de gravidade do triângulo oco.
Considere o incentro do triângulo cujos vértices são os pontos médios
dos lados do triângulo ABC. Este ponto é o baricentro do triângulo ABC oco.

Abraços,

E. Wagner.



Quoting Paulo Santa Rita paulo.santar...@gmail.com:


Oi Ponce e demais colegas
desta lista ... OBM-L,

EU PENSO que um possivel objetivo comum a todos nos e justamente o de
propor problemas interessantes, nao triviais, e/ou suas respectivas
solucoes, quando possiveis.  Procedendo assim PARECE-ME que ocorrerao
duas implicacoes obvias :

1) Serao naturalmente afastados aqueles(as) que vem esta lista como um
SOS para vestibulandos, concurseiros e assemelhados.
2) Serao naturalmente atraidos  aqueles(as) que aspiram ler e aprender
uma Matematica de mais alta qualidade.

E eu tenho certeza - ate por experiencia pessoal - que com a
colaboracao das inumeras pessoas talentosas que ja passaram por aqui,
podemos criar um centro de discussao de Matematica Elementar que nao
ficara devendo a nenhum outro centro do Mundo

Quando ao problema do Nehab, e bastante razoavel supor que o campo
gravitacional e constante ( pois estamos imaginando o triangulo
proximo a superficie da terra). Agora, basta aplicar o teorema de
Varignon.

Seja ABC um triangulo. Sem perda de generalidade podemos supor que os
seus vertices estao disposto tais que A esta em (0,0), B em (c,0) e C
em (X,Y). E natural supor que a densidade linear de massa e constante
em cada lado, donde decorre que os respectivos centros de massa dos
seus lados estarao nos pontos medios destes lados. Seja d esta
densidade liner. As forcas que atuam em cada ponto medio sao :

No lado AB, cdg, atuando em (c/2,0)   -  aqui,  g e a aceleracao da gravidade
No lado AC,  bdg, atuando em (X/2,Y/2)
No lado BC, adg, atuando em ( (c+X)/2 , Y/2 )

Assim VERTICALMENTE, temos uma forca de cdg em 0, uma forca bdg em Y/2
e uma forca adg em Y/2, vale dizer, uma forca, (bdg+adg) em Y/2. A
forca que equilibra tudo isso tem modulo, obviamente, igual adg + bdg
+ cdg atuando na posicao vertical m ( Varignon agora ) tal que :

m*(adg + bdg + cdg) = cdg*0 + (bdg + adg)*(Y/2)
m =( (a+b) / (a+b+c) )*(Y/2)   Resultado 1

HORIZONTALMENTE, temos uma forca cdg em c/2, uma forca bdg em (X/2) e
uma forca adg atuando em (c+X)/2. Para equilibrar isso devemos ter uma
forca de modulo  obviamente, igual adg + bdg + cdg e atuando na
posicao horizontal n ( Varignon agora ) tal que :

n*(adg + bdg + cdg)  = cdg*(c/2) + bdg*(X/2) + adg*( (c+X)/2)
n = (  (a+c)*(c/2) + (b+c)*(X/2)  ) / (a + b + c) Resultado 2

Os dados em Resultado 1 e 2 sao conhecidos, bastando construir o
sistema de coordenadas indicado ( traca-se por A uma perpendicular a
AB, Esta perpendicular e o eixo Y; o eixo X sera a reta que suporta o
lado AB). Os valores de m e n sao facilmente construtiveis. Logo:

O PONTO PROCURADO

Traca-se por m uma paralela ao eixo X e por n uma paralela ao eixo Y.
O ponto de encontro e o ponto procurado.

Um Abracao a Todos !
PSR,11005091409


pois isto afastara aqueles estudantes preguicosos ,  para que aqueles
que nos acompanham
2009/5/10 Rogerio Ponce abrlw...@gmail.com:

Ola'  Santa Rita,
obrigado pelo elogio.
Mas o culpado foi voce mesmo, ao trazer problemas interessantes   
para a lista.

Parabens!

Alias, o Nehab e' outro provocador  - vide o probleminha que ele
propos (e que eu reescrevo abaixo):


Dado o desenho de uma armacao triangular, feita de arame extremamente
fino (unidimensional), encontre o seu centro de gravidade,
utilizando-se apenas de desenho geometrico.
(ou seja, a solucao tem que ser tracada, nao podendo ser apenas
expressa por meio de equacoes.)

Abracos a todos,
Rogerio Ponce



2009/5/9 Paulo Santa Rita paulo.santar...@gmail.com:

Ola Ponce, Nehab, Luis Lopes
e demais colegas desta lista ... OBM-L,

Ponce, a sua solucao, simples e bela, e tipica de uma Matematica de
qualidade. Parabens por ela !
Fico feliz por ter iniciado uma discussao que lhe interessou, trouxe o
Ralph, o Shine e levou outros Matematicos de qualidade a se
manifestarem.

ENTRE MUITO OUTROS Matematicos de qualidade que outrora apareciam por
aqui e que ja ha algum tempo nao escrevem, sem duvida se incluem o
Nicolau e o Gugu. Oxala eles voltem a escrever brevemente !

Uma questao que sempre me interessou, subsidiariamente, e a seguinte :

Sabemos que o INCENTRO ( Centro do Circulo inscrito a um triangulo )
nao faz parte da reta de Euler, isto e, ele nao esta NECESSARIAMENTE
alinhado com os pontos notaveis que pertencem a esta reta. Assim em
geral, o incentro, o circuncentro e o ortocentro formam um pequeno
triangulo no interior de um triangulo dado. O que se pode falar sobre
esse pequeno triangulo ? Que relacao ele mantem com o triangulo
original ?

Um abraco a Todos !
PSR,7090509132D

=
Instruções para entrar na lista, sair da lista e

Re: [obm-l] Wiki! Wiki! Wiki!

2008-02-08 Por tôpico wagner

Parabéns, Tengam,
está excelente e vai melhorar mais com as contribuições de todos.

Abraço,

E. Wagner.



Quoting Eduardo Tengan [EMAIL PROTECTED]:


Olá,

estou escrevendo para divulgar a nossa wiki de olimpíada, a Olimpédia,
que foi montada num computador gentilmente cedido pelo prof. Yoshiharu
Kohayakawa do IME/USP (Yoshi para os íntimos).  Acesse

 http://erdos.ime.usp.br

Inscrevam-se e divirtam-se!  A idéia desta wiki é criar uma referência
centralizada e de fácil acesso aos fatos matemáticos mais utilizados
nas diversas olimpíadas, em seus vários níveis.

Veja que não é necessário escrever tudo a partir do zero, a wiki
possui facilidades que permitem, por exemplo, criar links para a
página da OBM referenciando artigos da revista Eureka!  Obviamente não
queremos criar uma wiki de links apenas, mas sim construir um guia de
referência prática para ajudar não só aqueles que estão se preparando
para as diversas competições matemáticas mas também os demais
simpatizantes da Matemática Olímpica!

Por enquanto, a Olimpédia está bem vazia, aguardando a contribuição de
pessoas como você, participante da lista da OBM.  Alguns temas foram
sugeridos com base no conteúdo das últimas provas do nível 3 da OBM,
da IMO e da Ibero.  Está estudando raízes primitivas?  A desigualdade
de Cauchy-Schwartz?  Por que não criar uma página na Olimpédia a
respeito?  Afinal de contas, você só pode dizer que aprendeu mesmo um
assunto quando for capaz de explicá-lo a outra pessoa!

Veja que a Olimpédia está aberta aos outros níveis também, desde o
nível 1 até o universitário.  Nada impede que você crie, por exemplo,
uma página com título assuntos para iniciantes ou temas quentes
para a cone-sul ou o que devo saber para a Putnam.
Aqui contamos também com a experiência dos
professores que ajudam com o treinamento, com sugestões para tais
temas.  Mas se você tem sua própria lista de temas, vá em frente e
escreva!  A wiki é um esforço colaborativo e coletivo, e o seu formato
não é pré-determinado por um grupo de notáveis olímpicos, mas sim
por seus diversos usuários.  Além de ser mais um mero usuário da wiki,
o meu papel, como dos demais administradores da wiki, é apenas de
coibir os famigerados off-topics e garantir que não haja desvios de
seus propósitos iniciais, que é, reitero, de servir como referência
para os fatos matemáticos mais úteis nas diversas competições
matemáticas.  Para demais temas, existem outros canais mais
apropriados, aqui mesmo na internet.

Bem, é isso!  Espero seu artigo em breve!

Até,
ET





=


--
Powered by Outblaze

=
Instruções para entrar na lista, sair da lista e usar a lista em
http://www.mat.puc-rio.br/~obmlistas/obm-l.html
=






This message was sent using IMP, the Internet Messaging Program.



=
Instruções para entrar na lista, sair da lista e usar a lista em
http://www.mat.puc-rio.br/~obmlistas/obm-l.html
=


Re: [obm-l] Geometria Esferica

2007-05-03 Por tôpico Eduardo Wagner
Uma pergunta off mas pertinente:
Quem assina esta mensagem é o famoso professor Zoroastro,
que fez a sensacional demonstração do Teorema de Euler dos
poliedros e pai de Zoroastro Azambuja Filho, nosso também
importante jovem matemático?

--
From: Zoroastro Azambuja [EMAIL PROTECTED]
To: obm-l@mat.puc-rio.br
Subject: [obm-l] Geometria Esferica
Date: Thu, May 3, 2007, 9:00 PM


Alguém poderia me dizer quais são os axiomas válidos na geometria esferica
(ou eliptica)?



obrigado.
 __
Fale com seus amigos  de graça com o novo Yahoo! Messenger
http://br.messenger.yahoo.com/


Re: [obm-l] RES: [obm-l] Livro de Á lgebra Linear

2007-03-27 Por tôpico Eduardo Wagner
Algebra Linear, Elon Lages Lima, SBM.

--
From: Artur Costa Steiner [EMAIL PROTECTED]
To: obm-l@mat.puc-rio.br
Subject: [obm-l] RES: [obm-l] Livro de Álgebra Linear
Date: Tue, Mar 27, 2007, 6:31 PM


Um excelente eh o do Serge Lang, Linear Algebra
-Mensagem original-
De: [EMAIL PROTECTED] [mailto:[EMAIL PROTECTED] nome de
Leonardo Borges Avelino
Enviada em: terça-feira, 27 de março de 2007 17:14
Para: obm-l
Assunto: [obm-l] Livro de Álgebra Linear

Gostaria d saber bons livros d álgebra linear em teoria...




Re: [obm-l] Uma Curva Interessante

2006-02-07 Por tôpico Eduardo Wagner
Oi Paulo:
Oi Pessoal:

Esta eh facil. Faça um primeiro trecho quase, mais 
quase mesmo, horizontal e o restante do comprimento
caindo abruptamente até B. Como nesse primeiro trecho
a inclinação pode ser tão pequena quanto se queira,
o tempo que a bolinha demora para percorre-lo pode ser
arbitrariamente grande.
Logo, não existe a curva do tempo máximo.

Abraços,

Wagner.

--
From: Paulo Santa Rita [EMAIL PROTECTED]
To: obm-l@mat.puc-rio.br
Subject: [obm-l] Uma Curva Interessante
Date: Tue, Feb 7, 2006, 2:14 PM


 Ola Pessoal,

 Alguem me propos o seguinte problema :

 Dentre todas as curvas de mesmo comprimento L que ligam dois pontos A e B
 de um plano, determinar aquela em que um corpo submetido exclusivamente ao
 campo gravitacional da terra (suposto constante ) gasta o tempo maximo para
 ir de A para B.

 NOTA : Se A=(Xa,Ya) e B=(Xb,Yb) sao as coordenadas de A e B suponha que Xb 
 Xa e
 Yb  Ya. Tambem suponha que :

 distancia entre A e BL   (Xb - Xa) + (Ya - Yb)

 Parece ser um problema interessante, nao trivial. Como estou sem tempo pra
 pensar nele, estou passando pra voces.

 Um Abraco a Todos
 Paulo Santa Rita
 3,1414,070206

 _
 Facilite sua vida: Use o Windows Desktop Search e encontre qualquer arquivo
 ou e-mail em seu PC. Acesse:  http://desktop.msn.com.br

 =
 Instruções para entrar na lista, sair da lista e usar a lista em
 http://www.mat.puc-rio.br/~nicolau/olimp/obm-l.html
 =

=
Instruções para entrar na lista, sair da lista e usar a lista em
http://www.mat.puc-rio.br/~nicolau/olimp/obm-l.html
=


Re: [obm-l] lidski

2005-10-30 Por tôpico Eduardo Wagner
Nunca soube que tenha sido publicado aqui.
O meu Lidski eh publicado pela MIR em 1972
na edicao em espanhol.

--
From: marcio aparecido [EMAIL PROTECTED]
To: obm-l@mat.puc-rio.br
Subject: [obm-l] lidski
Date: Fri, Oct 28, 2005, 5:38 PM


 Oieee galera
 queria pergintar se alguem aqui da lista tem o lidski, e em que línguas
 ele foi publicado aqui no brasil ??

 =
 Instruções para entrar na lista, sair da lista e usar a lista em
 http://www.mat.puc-rio.br/~nicolau/olimp/obm-l.html
 =

=
Instruções para entrar na lista, sair da lista e usar a lista em
http://www.mat.puc-rio.br/~nicolau/olimp/obm-l.html
=


Re: [obm-l] sistemas lineares

2005-07-16 Por tôpico Eduardo Wagner
Title: Re: [obm-l] sistemas lineares



Michele:
Em primeiro lugar se voce examinar a demonstracao da
regra de Cramer, voce vera que o resultado so vale se 
o determinante do sistema for diferente de zero. A regra
de Cramer, portanto, nao se dedica a discutir nada.
Em segundo lugar, mesmo que algumas pessoas insistam
em discutir um sistema linear usando os tais determinantes
Dx, Dy, etc, elas devem saber que a conclusao pode ser falsa.
Por exemplo, considere o simples sistema:
x + y + z = 1
2x + 2y + 2z = 3
3x + 3y + 3z = 5
Neste sistema, D = 0, Dx = Dy = Dz = 0. Os que usam erradamente
a regra de Cramer para discutir sistemas devem dizer que este
sistema eh indeterminado. Mas nao eh. Este sistema eh impossivel!
Em terceiro lugar, determinante eh coisa muito pouco pratica.
Quando o sistema tem 3 incognitas, ainda se admite que se possa
usar determinantes para resolver, mas, na vida real, sistemas 
lineares costumam ser muito maiores. Engenharia eletrica e Economia
sao areas que costumam lidar com sistemas grandes. E ninguem
eh doido o suficiente para pensar em usar determinantes.
Ha algum tempo, um conhecido meu do IMPA calculou o tempo
que um computador comum como o meu ou o seu levaria para
calcular um determinante 20X20. E o resultado foi: 1 ano,
1 mes e 17 dias.

Abraco,

W.


--
From: Michele Calefe [EMAIL PROTECTED]
To: obm-l@mat.puc-rio.br
Subject: Re: [obm-l] sistemas lineares
Date: Sat, Jul 16, 2005, 2:26 PM


Eduardo, mas quando o sistema tem o número de incógnitas igual ao número de equações, e, o determinante é zero, dá pra dizer que se todos os Dx, Dy,...forem nulos, o sistema é SPI? Além disso, se pelo menos um deles é diferente de zero o sistema é SI? Por que não faz sentido discutir dessa maneira?
michele

Eduardo Wagner [EMAIL PROTECTED] escreveu:
MIchele:

A regra de Cramer eh um metodo que permite
explicitar cada incognita de um sistema linear com
mesmo numero de equacoes e incognitas quando o
determinante do sistema eh diferente de zero.
Tem interesse teorico mas, na pratica eh terrivelmente
ineficiente.
A regra de Cramer nao serve para discutir sistemas.
A melhor forma de discutir um sistema linear com m
equacoes e n incognitas eh o escalonamento.

Abraco.

W.

--
From: Michele Calefe [EMAIL PROTECTED]
To: obm-l@mat.puc-rio.br
Subject: [obm-l] sistemas lineares
Date: Fri, Jul 15, 2005, 3:52 PM


Pessoal, eu gostaria de saber se é possível discutir um sistema linear utilizando a regra de Cramer. Sei que não é possível encontrar a solução do SPI, mas, é possível afirmar quando o sistema é SI ou SPI?

obrigada,

michele
__
Converse com seus amigos em tempo real com o Yahoo! Messenger 
http://br.download.yahoo.com/messenger/ 


Yahoo! Acesso Grátis  http://us.rd.yahoo.com/mail/br/taglines/*http://br.acesso.yahoo.com/ : Internet rápida e grátis. Instale o discador agora!  http://us.rd.yahoo.com/mail/br/taglines/*http://br.acesso.yahoo.com/ 





Re: [obm-l] sistemas lineares

2005-07-15 Por tôpico Eduardo Wagner
Title: Re: [obm-l] sistemas lineares



MIchele:

A regra de Cramer eh um metodo que permite
explicitar cada incognita de um sistema linear com
mesmo numero de equacoes e incognitas quando o
determinante do sistema eh diferente de zero.
Tem interesse teorico mas, na pratica eh terrivelmente
ineficiente.
A regra de Cramer nao serve para discutir sistemas.
A melhor forma de discutir um sistema linear com m
equacoes e n incognitas eh o escalonamento.

Abraco.

W.

--
From: Michele Calefe [EMAIL PROTECTED]
To: obm-l@mat.puc-rio.br
Subject: [obm-l] sistemas lineares
Date: Fri, Jul 15, 2005, 3:52 PM


Pessoal, eu gostaria de saber se é possível discutir um sistema linear utilizando a regra de Cramer. Sei que não é possível encontrar a solução do SPI, mas, é possível afirmar quando o sistema é SI ou SPI?

obrigada,

michele
__
Converse com seus amigos em tempo real com o Yahoo! Messenger 
http://br.download.yahoo.com/messenger/ 





Re: [obm-l] Geracao de ciclos , qual eh a logica?

2005-04-18 Por tôpico Wagner Ferreira
Escreva o conjunto de índices acima e abaixo a
permutação dada.
(0,1,2,3,4,5,6)
(2,5,4,0,3,1,6)

A lógica é a seguinte:
Inicie pelo 0 e observe quem está na posição do 0 na
permutação, no caso 2. Então você observará quem fica 
na posição do 2 na permutação: 4. Em seguida, na
posição do 4 temos o 3. Como na posição do 3 está o 0
então fechamos o primeiro ciclo. O segundo ciclo é só
tomar um valor que não tenha aparecido no primeiro e
seguir os passos descritos anteriormente. Então terá:

Ciclo 2: 1, 5 
Ciclo 3: 6

Uma coisa interessante é fazer o trabalho inverso,
isso é, dados os ciclos determinar a permutação.
Por exemplo: dados

ciclo 1: 1,3,5
ciclo 2: 2,4,0

temos a permutação P=(2,3,4,5,0,1)

Falou!
Wagner (Bug)

__
Converse com seus amigos em tempo real com o Yahoo! Messenger 
http://br.download.yahoo.com/messenger/ 
=
Instruções para entrar na lista, sair da lista e usar a lista em
http://www.mat.puc-rio.br/~nicolau/olimp/obm-l.html
=


Re: [obm-l] Mais construcoes [era: Quadrilatero Inscritivel]

2004-11-10 Por tôpico Eduardo Wagner
Oi Luiz e amigos da lista:

1) A solucao que conhecia do quadrilatero inscritivel
eh a mesma do livro do Natan.
2) Para os amigos da lista que nao entenderam nada do
comentario de Luiz Lopes sobre Petersen explico:
Julius Petersen foi um personagem do inicio do sec.20
que escreveu um livro sobre construcoes geometricas que
nao tem uma unica figura. Eh muito dificil de entender.
Dai o seu comentario sobre expert.
3) Eu sei fazer o problema que Luiz propos tirado do
fundo do bau. Mas, eh claro, nao vou mandar a solucao
agora.

Abracos,

Wagner.

--
From: Luís Lopes [EMAIL PROTECTED]
To: [EMAIL PROTECTED]
Subject: Re: [obm-l] Mais construcoes [era: Quadrilatero Inscritivel]
Date: Wed, Nov 10, 2004, 3:34 PM


 Sauda,c~oes,

 Oi Wagner,

Declaro resolvida a questao do quadrilatero inscritivel.
 Qual seria a sua solução? A mesma? Pesquisando ontem no
 Petersen ele apresenta (ou melhor, sugere) uma mas não
 entendi, como foi quase sempre o caso nas soluções desse livro.

Para os que nao conhecem, Luiz Lopes eh um expert em construcoes
geometricas.
 Obrigado pelo elogio mas experts são aqueles que conseguem entender
 e reproduzir as soluções do Petersen. Ou bolar outras para os
 problemas que ele apresenta. Ou para este aqui, tirado de
 Alexandroff (Aleksandrov), Ivan, Problèmes de Géométrie Élémentaire,
 Hermann, Paris, 1899 (mais do fundo do baú ainda!!! :))

 Construir um quad. ABCD dados os ângulos e as diagonais.

Ele eh um excelente matematico e publicou varios livros
sobre diversos assuntos. Um deles se chama
Manual de construcao de Triangulos que eh uma verdadeira
preciosidade.
 Este livro foi publicado em francês e está esgotado. Ah, não
 foi best seller não, só imprimi 40 exemplares. Pretendo publicá-lo
 em português também, ocasião em que farei diversas alterações
 e apresentarei soluções que me escaparam. Algumas
 delas por falta de uma investigação mais intensa mas outras
 somente após consultar um livro em alemão que me foi oferecido
 recentemente por um membro de uma outra lista.

 []'s
 Luis


From: Eduardo Wagner [EMAIL PROTECTED]
Reply-To: [EMAIL PROTECTED]
To: [EMAIL PROTECTED]
Subject: Re: [obm-l] Mais construcoes [era: Quadrilatero Inscritivel] Date:
Tue, 09 Nov 2004 23:42:35 -0200

Declaro resolvida a questao do quadrilatero inscritivel.
Para os que nao conhecem, Luiz Lopes eh um expert em construcoes
geometricas. Ele eh um excelente matematico e publicou varios livros
sobre diversos assuntos. Um deles se chama
Manual de construcao de Triangulos que eh uma verdadeira
preciosidade.
Vai ser dificil achar um livro sobre o assunto que ele ainda
nao tenha, mas vou procurar descobrir.
Abracos,
Wagner.


--
 From: Luís Lopes [EMAIL PROTECTED]
 To: [EMAIL PROTECTED]
 Subject: [obm-l] Mais construcoes [era: Quadrilatero Inscritivel]
 Date: Tue, Nov 9, 2004, 6:41 PM
 

  Sauda,c~oes,
 
  Oi Claudio,
 
  ===
 O problema estah morto e acho que voce acabou de ganhar um livro do
Eduardo
 Wagner.
  ===
  Poderia ser o caso se não tivesse enviado a solução de
  Altshiller-Court, Nathan, College Geometry, 1952.
 
  Talvez esse problema esteja no FG-M também. Não olhei.
 
  As primeiras tentativas de solução da lista para este problema
  baseavam-se na construção de elementos obtidos algebricamente
  (diagonais e circumraio, se me lembro bem).
  Pergunto: tendo-se mostrado que o problema tem uma solução
  algébrica, será que SEMPRE podemos obter uma solução
  geométrica? Penso que sim, depois de ver soluções
  geométricas para muitos problemas onde achava que só a
  solução bruta algébrica seria possível.
 
  Proponho então dois problemas para os quais tenho somente
  sols. algébricas. Será que existiriam sols. geom. também???
 
  Construir o triângulo ABC dados:
 
  1) A, m_a, r
  2) A, m_a, r_a
 
  A=ângulo, m_a = mediana que parte de A; r (in-raio) r_a (ex-raio).
 
  Amanhã proponho mais um de quadrilátero.
 
  []'s
  Luis
 


 =
 Instruções para entrar na lista, sair da lista e usar a lista em
 http://www.mat.puc-rio.br/~nicolau/olimp/obm-l.html
 =

=
Instruções para entrar na lista, sair da lista e usar a lista em
http://www.mat.puc-rio.br/~nicolau/olimp/obm-l.html
=


Re: [obm-l] Mais construcoes [era: Quadrilatero Inscritivel]

2004-11-09 Por tôpico Eduardo Wagner
Declaro resolvida a questao do quadrilatero inscritivel.
Para os que nao conhecem, Luiz Lopes eh um expert em construcoes
geometricas. Ele eh um excelente matematico e publicou varios livros
sobre diversos assuntos. Um deles se chama
Manual de construcao de Triangulos que eh uma verdadeira
preciosidade.
Vai ser dificil achar um livro sobre o assunto que ele ainda
nao tenha, mas vou procurar descobrir.
Abracos,
Wagner.


--
From: Luís Lopes [EMAIL PROTECTED]
To: [EMAIL PROTECTED]
Subject: [obm-l] Mais construcoes [era: Quadrilatero Inscritivel]
Date: Tue, Nov 9, 2004, 6:41 PM


 Sauda,c~oes,

 Oi Claudio,

 ===
O problema estah morto e acho que voce acabou de ganhar um livro do Eduardo
Wagner.
 ===
 Poderia ser o caso se não tivesse enviado a solução de
 Altshiller-Court, Nathan, College Geometry, 1952.

 Talvez esse problema esteja no FG-M também. Não olhei.

 As primeiras tentativas de solução da lista para este problema
 baseavam-se na construção de elementos obtidos algebricamente
 (diagonais e circumraio, se me lembro bem).
 Pergunto: tendo-se mostrado que o problema tem uma solução
 algébrica, será que SEMPRE podemos obter uma solução
 geométrica? Penso que sim, depois de ver soluções
 geométricas para muitos problemas onde achava que só a
 solução bruta algébrica seria possível.

 Proponho então dois problemas para os quais tenho somente
 sols. algébricas. Será que existiriam sols. geom. também???

 Construir o triângulo ABC dados:

 1) A, m_a, r
 2) A, m_a, r_a

 A=ângulo, m_a = mediana que parte de A; r (in-raio) r_a (ex-raio).

 Amanhã proponho mais um de quadrilátero.

 []'s
 Luis


From: Claudio Buffara [EMAIL PROTECTED]
Reply-To: [EMAIL PROTECTED]
To: [EMAIL PROTECTED]
Subject: Re: [obm-l] Quadrilatero Incritivel
Date: Tue, 09 Nov 2004 17:36:58 -0200

on 08.11.04 12:35, Luís Lopes at [EMAIL PROTECTED] wrote:
 
  Sejam AB=a, BC=b, CD=c, DA=d, AC=x, BD=y e seja m a reta
  simétrica do lado AD com relação à bissetriz do ângulo BAC.
 
  Lema: a reta m contém um e somente um ponto O tal que o /_ AOB = /_ ACD
.
  O ponto O \in m pertence ao lado BC sss ABCD é cíclico.
 
Agora faz sentido!

  Dos triângulos ACD e AOB, temos /_ ABO = /_ ADC .
 
  Assim, se ABCD é cíclico, o ponto O está no lado BC; e somente nesse
caso,
  pois, reciprocamente, se O está em BC então ABCD é cíclico.
 
  Teorema: (Ptolomeu) xy = ac + bd sss ABCD é cíclico.
 
  Na dem. do lema acima mostra-se que OB = ac/d e que AO/AC = a/d.
 
Pois os triangulos OBA e CDA sao semelhantes.

  Daí a const. que segue:
 
  1) Numa reta r marque CB = b e construa O tal que BO = ac/d
  (com B entre O e  C). Isso implica que OC = (ac + bd)/d = xy/d.
 
  2) um lg para A é o círculo (B,a). O outro é um círc. de Apolônio
  considerando os pontos O e C.
 
Ou seja, A pertence ao l.g. dos pontos X tais que |XO|/|XC| = a/d.

Legal, com A construido, basta tracar os circulos (A,d) e (C,c), cujo ponto
de interseccao no interior do angulo ABC eh justamente D.

O problema estah morto e acho que voce acabou de ganhar um livro do Eduardo
Wagner.

[]s,
Claudio.



 =
 Instruções para entrar na lista, sair da lista e usar a lista em
 http://www.mat.puc-rio.br/~nicolau/olimp/obm-l.html
 =

=
Instruções para entrar na lista, sair da lista e usar a lista em
http://www.mat.puc-rio.br/~nicolau/olimp/obm-l.html
=


Re: [obm-l] Olimpíadas Brasileiras de Matemática, 1ª a 8ª

2004-11-01 Por tôpico Eduardo Wagner
Eu tenho Fabio.
Entre em contato comigo [EMAIL PROTECTED].

--
From: Fabio Dias Moreira [EMAIL PROTECTED]
To: [EMAIL PROTECTED]
Subject: Re: [obm-l] Olimpíadas Brasileiras de Matemática, 1ª a 8ª
Date: Sat, Oct 30, 2004, 11:48 AM



 machado said:
 Olá pessoal,
 alguém tem esse livro ? Olimpíadas Brasileiras de Matemática, 1ª a 8ª -
 Élio Mega e Renate Watanabe ?

 Preciso muito.
 Serve xerox ou original.
 [...]

 Veja o site da SBM:

 http://www.sbm.org.br/

 []s,

 --
 Fábio ctg \pi Dias Moreira


 =
 Instruções para entrar na lista, sair da lista e usar a lista em
 http://www.mat.puc-rio.br/~nicolau/olimp/obm-l.html
 =

=
Instruções para entrar na lista, sair da lista e usar a lista em
http://www.mat.puc-rio.br/~nicolau/olimp/obm-l.html
=


Re: [obm-l] Quadrilatero Incritivel

2004-10-29 Por tôpico Eduardo Wagner
Vamos com calma minha gente.
Para os que nao estao habituados com as construcoes
geometricas em toda a sua pureza e sofisticacao,
o enunciado do problema eh o seguinte:

Na sua folha de papel existem 4 segmentos que sao os
lados AB, BC, CD e DA do quadrilatero inscritivel ABCD.
Pede-se desenhar o quadrilatero possuindo uma regua nao
graduada e compasso.
OBS: Para facilitar, admite-se o uso de esquadros para
o tracado de paralelas e perpendiculares. Na verdade,
esses instrumentos nao sao necessarios pois com regua
nao graduada e compasso podemos fazer essas construcoes.

Engana-se o colega que afirma que com uma pesquisa se pode
responder a qualquer coisa. Neste caso, estamos propondo
um problema onde nao ha nenhum instrumento de medida.
Formulas sao permitidas, eh claro, mas devem ser construidas
com regua nao graduada e compasso para terem validade.

Para saber mais: As primeiras propriedades do quadrilatero
inscritivel apareceram no livro Almajesto de Ptolomeu
no sec.2 dC. O problema em questao, de construir um
quadrilatero inscritivel conhecendo os 4 lados nao tem pai,
mas foi objeto de atencao por muita gente durante muito tempo.
A primeira solucao foi dada por Francois Viete em 1540.

Abracos,

Wagner.


--
From: Qwert Smith [EMAIL PROTECTED]
To: [EMAIL PROTECTED]
Subject: Re: [obm-l] Quadrilatero Incritivel
Date: Thu, Oct 28, 2004, 4:53 PM


From: Eduardo Wagner [EMAIL PROTECTED]
 From: Claudio Buffara [EMAIL PROTECTED]
  Aqui vai um problema proposto ha tempos pelo Eduardo Wagner e que nunca
foi
  resolvido na lista:
 
  Construir um quadrilatero inscritivel ABCD dados AB e os comprimentos de
BC,
  CD e DA.
 
  []s,
  Claudio.
 
Boa Claudio! Vamos botar esse pessoal para pensar. Como eh um problema de
enunciado absolutamente lindo vou oferecer um livro para a primeira solucao
correta que aparecer na lista.

Abracos a todos.

 Nos dias de hj qualquer um responde qualquer pergunta com uma simples
 pesquisa
 na net.

 Veja o link:
 http://mathworld.wolfram.com/CyclicQuadrilateral.html

 Ele te da a formula das diagonais do quadrilatero incritivel em funcao dos
 lados.
 Ai vc faz assim:
 de A traca com um compasso a diagonal AC, de B traca o lado BC
 a intercao eh o ponto C.
 O ponto D eh analogo e acabou.

 Uma resposta quase 'Dirichlet'lesca.
 []s,
 Auggy

 _
 Express yourself instantly with MSN Messenger! Download today - it's FREE!
 http://messenger.msn.click-url.com/go/onm00200471ave/direct/01/

 =
 Instruções para entrar na lista, sair da lista e usar a lista em
 http://www.mat.puc-rio.br/~nicolau/olimp/obm-l.html
 =

=
Instruções para entrar na lista, sair da lista e usar a lista em
http://www.mat.puc-rio.br/~nicolau/olimp/obm-l.html
=


Re: [obm-l] Quadrilatero Incritivel

2004-10-28 Por tôpico Eduardo Wagner


--
From: Claudio Buffara [EMAIL PROTECTED]
To: Lista OBM [EMAIL PROTECTED]
Subject: [obm-l] Quadrilatero Incritivel
Date: Fri, Jan 1, 1904, 12:37 PM


 Aqui vai um problema proposto ha tempos pelo Eduardo Wagner e que nunca foi
 resolvido na lista:

 Construir um quadrilatero inscritivel ABCD dados AB e os comprimentos de BC,
 CD e DA.

 []s,
 Claudio.

Boa Claudio! Vamos botar esse pessoal para pensar. Como eh um problema de
enunciado absolutamente lindo vou oferecer um livro para a primeira solucao
correta que aparecer na lista.

Abracos a todos.
 =
 InstruÁžes para entrar na lista, sair da lista e usar a lista em
 http://www.mat.puc-rio.br/~nicolau/olimp/obm-l.html
 =

=
Instruções para entrar na lista, sair da lista e usar a lista em
http://www.mat.puc-rio.br/~nicolau/olimp/obm-l.html
=


Re: [obm-l] Classificação de Grupos

2004-07-03 Por tôpico Eduardo Wagner
Epa! Mensagens do Nicolau que sempre chegaram claras
estao chegando com simbolos ininteligiveis.
O que ocorre?

--
From: Nicolau C. Saldanha [EMAIL PROTECTED]
To: [EMAIL PROTECTED]
Subject: Re: [obm-l] Classificação de Grupos
Date: Sat, Jul 3, 2004, 6:36 PM


 On Sat, Jul 03, 2004 at 03:33:16PM -0300, Lista OBM wrote:
 estou com dificuldades em classificar todos os grupos de ordem at� 11.
 Gostaria de saber se algu�m poderia me ajudar.

 Al�m de um grupo c�clico de cada ordem, temos Z/(2) + Z/(2),
 Z/(4) + Z/(2), Z/(2) + Z/(2) + Z/(2), Z/(3) + Z/(3),
 os grupo diedrais de ordens 6, 8 e 10 e o grupo {+-1,+-i,+-j,+-k}
 dos quat�rnios. N�o � muito dif�cil provar que estes s�o os
�nicos.

 � mais interessante classificar os grupos de ordem 12: h� 5 deles.

 []s, N.

 =
 Instruções para entrar na lista, sair da lista e usar a lista em
 http://www.mat.puc-rio.br/~nicolau/olimp/obm-l.html
 =

=
Instruções para entrar na lista, sair da lista e usar a lista em
http://www.mat.puc-rio.br/~nicolau/olimp/obm-l.html
=


[obm-l] parabola

2004-02-18 Por tôpico Eduardo Wagner
Title: parabola





--
From: 
Date: Wed, Feb 18, 2004, 4:00 PM


Alguem sabe como se resolve:

- Seja um triângulo ABC cujos lados são tangentes a uma parabola. Prove que o circulo circunscrito ao triângulo passa pelo foco.


Vamos la. Voces vao ter que fazer uma figura para acompanhar.
Seja ABC o triangulo. Uma parabola de foco F eh tangente em M
ao lado BC e tangente em N e P aos prolongamentos dos lados
AB e AC, respectivamente.
A semi-reta FX eh o eixo da parabola.
Trace por A e B as semi-retas AY e BZ paralelas a FX.
Pelo teorema de Poncelet, ang(NBZ) = ang(CBF).
Como BZ e AY sao paralelas, ang(NBZ) = ang(NAY).
Pelo teorema de Poncelet, ang(NAY) = ang(BAY) = ang(PAF) 
= ang(CAF).
Se ang(CBF) = ang(CAF) entao F pertence a circunferencia
circunscrita ao triangulo ABC.

Abraco,

Wagner.






Yahoo! GeoCities: a maneira mais fácil de criar seu web site grátis! 





Re: [obm-l] Matrizes Inversiveis

2004-02-09 Por tôpico Eduardo Wagner


--
From: Nicolau C. Saldanha [EMAIL PROTECTED]
To: [EMAIL PROTECTED]
Subject: Re: [obm-l] Matrizes Inversiveis
Date: Mon, Feb 9, 2004, 1:31 PM


 On Sun, Feb 08, 2004 at 08:39:13PM -0200, Claudio Buffara wrote:
 Sua solucao me gerou outra duvida. Qual a grafia correta: inversivel ou
 invertivel ou ambas sao aceitaveis?

 Quase todo mundo fala e escreve inversível. Algumas pessoas,
 entre elas o Elon, falam e escrevem invertível, argumentando
 que a palavra vem do verbo inverter e portanto o 't' não tem pq
 virar um 's'. O argumento é discutível, pois dizemos conversível
 e reversível apesar dos verbos serem converter e reverter.

 []s, N.

A questao nao tem maior importancia, mas o Aurelio registra:
intertivel = o que se pode inverter.


 =
 Instruções para entrar na lista, sair da lista e usar a lista em
 http://www.mat.puc-rio.br/~nicolau/olimp/obm-l.html
 =

=
Instruções para entrar na lista, sair da lista e usar a lista em
http://www.mat.puc-rio.br/~nicolau/olimp/obm-l.html
=


Re: [obm-l] rea Lateral de Pir mide

2003-11-11 Por tôpico Eduardo Wagner
Title: Re: [obm-l] rea Lateral de Pirmide



Para mim chegou ilegivel.

--
From: Cludio \(Prtica\) [EMAIL PROTECTED]
To: [EMAIL PROTECTED]
Subject: [obm-l] rea Lateral de Pirmide
Date: Tue, Nov 11, 2003, 16:56


Oi, Pessoal:

O problema abaixo  baseado numa questo da Olimpada Paulista de Matemtica desse ano.

Dado um quadrado ABCD, de lado a, determine o lugar geomtrico dos vrtices das pirmides que tm ABCD como base e rea lateral constante e igual a S.
(a, S: reais positivos).

Um abrao,
Claudio.
[EMAIL PROTECTED];s+ [EMAIL PROTECTED],Ak9Yt{T~tkTtO'T~UI/'E4YDwl Ea2c/G xYR +c%w= [EMAIL PROTECTED],ucz]*sYXF]fcuXGuc)Y2 ]ffu]2*uc/uc+]cW]2T]l+]f1ucvuX]WFu+]X*u*]W/]WcY+A]c*YWvY!Y2,ucbucwY*F]fcu*{YWb]2+]2]2TY]WZYcYWcY*2]*u,Y*+]X*W]fZ]WF]2 uXcu*]WZY*W]c+Y],ucz]*sYXF]fcuXGuc)Y2 ]ffu]2*uc/uc+]cW]2T]l+]f1ucvuX]WFu+]X*u*]W/]WcY+A]c+YW,Y*z:[EMAIL PROTECTED]@[EMAIL PROTECTED]@ 






Re: [obm-l] teorema de dandelin

2003-08-26 Por tôpico Eduardo Wagner
Title: Re: [obm-l] teorema de dandelin



Epa! 
Um engano na mensagem abaixo. O certo eh:
a) se o plano for paralelo a uma das geratrizes obtemos uma
parabola.
b) se cortar apenas uma das partes do duplo cone obtemos uma
elipse (ou uma circunferencia)
c) se cortar as duas partes do duplo cone obtemos uma hiperbole.

--
From: Artur Costa Steiner [EMAIL PROTECTED]
To: [EMAIL PROTECTED]
Subject: [obm-l] teorema de dandelin
Date: Mon, Aug 25, 2003, 6:09 PM



O Teorema de Dandelin eh aquela que trata das chamadas conicas, ou seja das curvas que se obtem ao se seccionar um cone reto com um plano. Se o plano for paralelo ao eixo do cone, obtemos uma parabola; se for paralelo a uma de suas geratrizes, obtemos uma hiperbole; e se cortar toda a supeficie lateral do cone, obtemos uma elipse (que eh um circulo se o plano for perpendicular ao eixo do cone). A demonstracao eh muito bonita e pode ser encontrada nos bons livros de geometria. 

Abracos
Artur


OPEN Internet
@ Primeiro provedor do DF com anti-vírus no servidor de e-mails @ = Instruções para entrar na lista, sair da lista e usar a lista em http://www.mat.puc-rio.br/~nicolau/olimp/obm-l.html = 





Re: [obm-l] BELEZA: belgas e pontos.

2003-08-14 Por tôpico Eduardo Wagner


--
From: Nicolau C. Saldanha [EMAIL PROTECTED]
To: [EMAIL PROTECTED]
Subject: Re: [obm-l]  BELEZA: belgas e pontos.
Date: Sun, Aug 10, 2003, 8:23 AM


 On Sun, Aug 10, 2003 at 02:08:56AM -0300, Eduardo Wagner wrote:
 Nao ha duvida sobre o que esta escrito acima. Entretanto, ha um pedaco de
 frase assim: e ja sabemos o que estas curvas sao.
 Sim! Nos sabemos, mas os alunos provavelmente nao.

 Como não? No ensino médio eles sabem. Eu estranho a idéia de estudar
 cônicas de forma puramente grega, sem mencionar que estas são as curvas
 de grau 2.

Eu nao disse isto. Os alunos conhecem as curvas do segundo grau mas
nao sao capazes de identificar as secoes no cone com elas uma vez que
a geometria analitica no espaco nao faz parte do programa de matematica
do ensino medio do nosso pais. Coordenadas no R3 so existe aqui no Rio de
Janeiro e, ainda assim, de forma muito elementar; equacao da reta e do
plano e nada mais.

 Ja que fui citado (ou provocado) em mensagem anterior, quero dizer que
 os alunos podem perfeitamente conhecer as conicas muito antes de estarem
 familiarizados com a geometria analitica no espaco, translacoes e rotacoes.
 E isto eh muito bom. Conhecer desde cedo as curvas e suas diversas formas,
 definidas por um unico numero chamado excentricidade.
 A demonstracao legal que usa as esferas eh totalmente elementar e permite
 obter um resultado surpreendente que vai agora para a beleza matematica
 da lista.

 Só para pacificar um pouco, eu também gosto muito da demonstração com
 as esferas, não estou de forma alguma querendo sugerir que a demonstração
 que apresentei (de natureza algébrica) deva eliminar a demonstração de
 natureza mais geométrica. Mas eu me lembro de, ao descobrir a demonstração
 algébrica, ter tido aquela sensação de isto é tão simples, pq me esconderam
 isso durante tanto tempo?...

Para pacificar totalmente eu talvez diria que voce deve ter tido bons
professores. Eles tiveram a nocao do que um aluno do ensino medio pode
compreender com as ferramentas que possuem na ocasiao, e deixar um mundo
de decobertas surpreendentes para depois, quando tiveram mais maturidade e
conhecimento.
E isto foi muito bom.

Abraco,

Wagner.


 []s, N.
 =
 Instruções para entrar na lista, sair da lista e usar a lista em
 http://www.mat.puc-rio.br/~nicolau/olimp/obm-l.html
 =
=
Instruções para entrar na lista, sair da lista e usar a lista em
http://www.mat.puc-rio.br/~nicolau/olimp/obm-l.html
=


Re: [obm-l] BELEZA: belgas e pontos.

2003-08-14 Por tôpico Eduardo Wagner


--
From: Nicolau C. Saldanha [EMAIL PROTECTED]
To: [EMAIL PROTECTED]
Subject: Re: [obm-l]  BELEZA: belgas e pontos.
Date: Sat, Aug 9, 2003, 10:37 PM


 On Sat, Aug 09, 2003 at 08:42:04PM -0300, Augusto Cesar de Oliveira Morgado
 wrote:
 Os teoremas a respeito de as seçoes do cone por planos que nao contem o
 vertice serem elipses, parabolas ou hiperboles foram demonstrados por dois
 belgas, Quetelet e Dandelin, e sao conhecidos por muitos como os teoremas
 belgas. As demonstraçoes sao particularmente elegantes e surpreendentemente
 simples, principalmente se expostas no quadro-negro pelo Wagner.

 Ok. A demonstração do Wagner deve ser uma que envolve desenhar umas esferas.
 É legal, mas não é minha demonstração favorita deste fato.
 Por mim a demonstração certa consiste em observar que o cone tem equação
 de grau 2 (x^2 + y^2 = z^2) e rodar ou transladar não altera o grau.
 Tomar a interseção com um plano, digamos o plano z=0, já que rodamos,
 também não altera o grau (só estamos eliminando os termos que envolvem z).
 Logo a interseção é uma curva de grau 2 e já sabemos o que estas curvas são.

 Observe que assim também demonstramos que a interseção de um parabolóide
 ou um hiperbolóide com um plano também é uma cônica. Em particular,
 a interseção de um hiperbolóide de revolução de uma folha (x^2 + y^2 = 1 +
z^2)
 com um plano tangente tem um ponto duplo, logo é um par de retas.

 []s, N.

Nao ha duvida sobre o que esta escrito acima. Entretanto, ha um pedaco de
frase assim: e ja sabemos o que estas curvas sao.
Sim! Nos sabemos, mas os alunos provavelmente nao.

Ja que fui citado (ou provocado) em mensagem anterior, quero dizer que
os alunos podem perfeitamente conhecer as conicas muito antes de estarem
familiarizados com a geometria analitica no espaco, translacoes e rotacoes.
E isto eh muito bom. Conhecer desde cedo as curvas e suas diversas formas,
definidas por um unico numero chamado excentricidade.
A demonstracao legal que usa as esferas eh totalmente elementar e permite
obter um resultado surpreendente que vai agora para a beleza matematica
da lista.

Em um cone (duplo) as geratrizes fazem angulo X com o eixo do cone e um
plano corta esse cone fazendo angulo Y com o eixo. Entao a secao do plano
com o cone eh uma curva cuja excentricidade eh cosY/cosX.

Abracos.

E. Wagner.









 =
 Instruções para entrar na lista, sair da lista e usar a lista em
 http://www.mat.puc-rio.br/~nicolau/olimp/obm-l.html
 =
=
Instruções para entrar na lista, sair da lista e usar a lista em
http://www.mat.puc-rio.br/~nicolau/olimp/obm-l.html
=


[obm-l] Re: no subject

2003-08-14 Por tôpico Eduardo Wagner


--
From: 
Date: Mon, Aug 11, 2003, 5:06 PM


 Na minha opiniao o Porisma de poncelet e que e
 contra-intuitivo:como e que e que uma coisa tao
 bonita pode ter uma demonstraçao tao feia???

 Dois problemas que nao resolvi mas acho legais
 neste ponto de vista:
 1)Existe uma funçao continua apenas nos
 racionais?

Nao.

 2)Existe uma funçao continua apenas nos
 irracionais?

Sim. Se x = p/q (irredutivel com p e q inteiros, q  0),
seja f(x) = 1/q se x eh racional e f(x) = 0 se x eh irracional.




  --- Claudio Buffara
 [EMAIL PROTECTED] escreveu:  on
 10.08.03 00:50, Artur Costa Steiner at
 [EMAIL PROTECTED] wrote:

  Aproveito a oportunidade para perguntar:
 Existe alguma conclusao da
  matematica que vc considere contraria aa
 intuicao? Eu, por exemplo, acho um
  tanto contra intuitivo que o fato de f ser
 diferenciavel  em R e apresentar
  limite no infinito nao implique que f'
 apresente limite zero no infinito.
  Algumas pessoas acham contra intuitivo que a
 serie harmonica seja
  divergente.
  Artur
 
 Oi, Artur:

 Gostaria de ver que exemplos outras pessoas da
 lista vao dar, mas assim de
 bate-pronto eu diria que acho contra-intuitivo:

 1) que existam funcoes continuas em toda a reta
 mas sem derivada em nenhum
 ponto;

 2) o fato de, sendo a irracional, o conjunto {
 m + na ; m, n inteiros } ser
 denso em R;

 3) que Pi tenha alguma relacao com a soma dos
 inversos dos quadrados dos
 naturais;

 4) que um problema tao simples como o de 3
 corpos sujeitos a atracao
 gravitacional mutua possa ter uma solucao
 caotica;

 5) que um conjunto nao enumeravel possa ter
 medida nula;

 6) que exista uma bijecao entre R e R^2;

 7) a maioria dos resultados quase-milagrosos de
 analise complexa;

 8) que R possa ser bem-ordenado e que isso seja
 consequencia de um negocio
 tao intuitivo como o axioma da escolha.

 9) que o porisma de Poncelet nao possa ser
 provado apenas por geometria
 Euclidiana.

 Mas acho que todos esses sao pinto se
 comparados ao

 10) paradoxo de Banach-Tarski - voce pode
 decompor uma esfera do tamanho de
 uma ervilha em no maximo 5 pedacos e re-montar
 esses pedacos de modo a
 formar uma esfera do tamanho do Sol

 E com essa, vou dormir...

 Um abraco,
 Claudio.


 =
 Instruções para entrar na lista, sair da lista
 e usar a lista em

 http://www.mat.puc-rio.br/~nicolau/olimp/obm-l.html

 =

 ___
 Conheça o novo Cadê? - Mais rápido, mais fácil e mais preciso.
 Toda a web, 42 milhões de páginas brasileiras e nova busca por imagens!
 http://www.cade.com.br
 =
 Instruções para entrar na lista, sair da lista e usar a lista em
 http://www.mat.puc-rio.br/~nicolau/olimp/obm-l.html
 =
=
Instruções para entrar na lista, sair da lista e usar a lista em
http://www.mat.puc-rio.br/~nicolau/olimp/obm-l.html
=


Re: [obm-l] ENQUETE - BELEZA MATEMATICA

2003-08-10 Por tôpico Eduardo Wagner
Caros amigos:

A enquete sobre a beleza matematica ja produziu uma lista
grande de belos teoremas.
Mas ficou faltando um na minha opiniao;

o teorema de Euler dos poliedros convexos: V - A + F = 2.

Nao eh uma coisa linda e inesperada? Eu, quando tive contato
com esse resultado pela primeira vez, fiquei pasmo, fascinado
pela sua simplicidade e beleza.
Eh tambem surpreendente pelas demonstracoes erradas ou incompletas
que apareceram durante cerca de 200 anos de historia.

Abracos,

E. Wagner.

--
From: Claudio Buffara [EMAIL PROTECTED]
To: Lista OBM [EMAIL PROTECTED]
Subject: [obm-l] ENQUETE - BELEZA MATEMATICA
Date: Sat, Aug 9, 2003, 10:24 AM


 Caros colegas da lista:

 Gostaria de contar com sua participacao numa enquete sobre beleza
 matematica.

 O que eu precisao eh que cada um de voces me envie uma lista contendo algo
 como 5 a 10 problemas/teoremas que voces consideram os mais bonitos e cujas
 solucoes/demonstracoes sao as mais elegantes e/ou inusitadas e/ou
 engenhosas. Nao precisa incluir a solucao/demonstracao, apenas o enunciado.
 No entanto, se voce tiver em mente uma solucao/demonstracao especifica
 (entre varias existentes) nao deixe de mencionar pelo menos o metodo
 utilizado.

 A unica restricao eh que estes resultados devem ser de um nivel acessivel a
 um aluno normal de 2o. grau (ou seja, o Ultimo Teorema de Fermat e o Porisma
 de Poncelet estao fora, mas o caso n = 4 do UTF e a versao para triangulos
 do Porisma poderiam ser incluidos).

 Importante: os resultados devem ser acessiveis a um aluno normal de 2o.
 grau, mas nao necessariamente fazer parte do curriculo normal do 2o. grau.

 Tambem nao precisa responder hoje ou amanha ou mesmo na semana que vem. Acho
 que vale a pena pensar por um tempo e consultar a literatura - as vezes pode
 ter um resultado belissimo do qual voce simplesmente se esqueceu por nao
 encontra-lo ha muito tempo. As Eurekas sao uma otima referencia. O Proofs
 from the Book tambem, apesar de nem tudo lah ter nivel de 2o. grau.

 Se houver um numero suficiente de respostas, eu me comprometo a publicar uma
 compilacao dos problemas e teoremas mais votados.

 Desde jah a gradeco o interesse de quem quiser participar.

 Um abraco,
 Claudio.


 =
 InstruÁžes para entrar na lista, sair da lista e usar a lista em
 http://www.mat.puc-rio.br/~nicolau/olimp/obm-l.html
 =
=
Instruções para entrar na lista, sair da lista e usar a lista em
http://www.mat.puc-rio.br/~nicolau/olimp/obm-l.html
=


Re: [obm-l] Problemas da IMO

2003-07-14 Por tôpico Eduardo Wagner
Parabens Gugu!

Voce eh realmente f (no bom sentido).
Eu ja carimbei alguns na OIM, mas na IMO,
sequer tentei.

Um grande abraco,

E. Wagner.

--
From: [EMAIL PROTECTED]
To: [EMAIL PROTECTED]
Subject: [obm-l] Problemas da IMO
Date: Mon, Jul 14, 2003, 3:38 PM




 Prova da IMO retirada do Site http://www.mathlinks.go.ro/

 O Problema 1 é nois que mandou...


 First Day - 44th IMO 2003 Japan

 1. Let A be a 101-element subset of the set S={1,2,3,...,100}. Prove that
 there exist numbers t_1, t_2, ..., t_{100} in S such that the sets

 Aj = { x + tj | x is in A } for each j = 1, 2, ..., 100

 are pairwise disjoint.


 2. Find all pairs of positive integers (a,b) such that the number

 a^2 / ( 2ab^2-b^3+1) is also a positive integer.

 3. Given is a convex hexagon with the property that the segment connecting the
 middle points of each pair of opposite sides in the hexagon is  sqrt(3) / 2
 times the sum of those sides' sum.

 Prove that the hexagon has all its angles equal to 120.


 Second Day - 44th IMO 2003 Japan

 4. Given is a cyclic quadrilateral ABCD and let P, Q, R be feet of the
 altitudes from D to AB, BC and CA respectively. Prove that if PR = RQ then the
 interior angle bisectors of the angles  ABC and  ADC are concurrent on AC.

 5. Let x1 = x2 = ... = xn be real numbers, n2.

 a) Prove the following inequality:

 (sum  ni,j=1 | xi - xj | ) 2 = 2/3 ( n^2 - 1 )sum ni,j=1 ( xi - xj)^2

 b) Prove that the equality in the inequality above is obtained if and only if
 the sequence (xk) is an arithemetical progression.

 6. Prove that for each given prime p there exists a prime q such that n^p - p
 is not divisible by q for each positive integer n.



 -
 This mail sent through IMP: http://horde.org/imp/
 =
 Instruções para entrar na lista, sair da lista e usar a lista em
 http://www.mat.puc-rio.br/~nicolau/olimp/obm-l.html
 =
=
Instruções para entrar na lista, sair da lista e usar a lista em
http://www.mat.puc-rio.br/~nicolau/olimp/obm-l.html
=


Re: [obm-l] Fw: Lista OBM

2003-07-14 Por tôpico Eduardo Wagner
Title: Re: [obm-l] Fw: Lista OBM



O que eh isso minha gente?
A lista nao pode conter mensagens deste tipo!

--
From: J.Paulo roxer ´til the end [EMAIL PROTECTED]
To: [EMAIL PROTECTED]
Subject: [obm-l] Fw: Lista OBM
Date: Sun, Jul 13, 2003, 10:04 PM


Por q vc não sai da teoria,constrói alguma coisa útil com matemática e explica o q vc sabe em vez de enviar besteiras pra mim?

- Original Message - 
From: Sérgio Martins mailto:[EMAIL PROTECTED] 
To: [EMAIL PROTECTED] mailto:[EMAIL PROTECTED] 
Sent: Sunday, July 13, 2003 9:17 PM
Subject: Lista OBM

Cara, você é chato. E bobo. Feio. Cara de mamão.

___
Yahoo! Mail
Mais espaço, mais segurança e gratuito: caixa postal de 6MB, antivírus, proteção contra spam.
http://br.mail.yahoo.com/ 


Email.it, the professional e-mail, gratis per te: clicca qui http://www.email.it/cgi-bin/start?sid=3 

Sponsor:
Interessi alti, massima libertà. È Conto Arancio di ING Direct.
Clicca qui http://adv.email.it/cgi-bin/foclick.cgi?mid=660d=14-7 






Re: [obm-l] Re: [obm-l] RE: [obm-l] Paulo e suas ...

2003-07-14 Por tôpico Eduardo Wagner
Title: Re: [obm-l] Re: [obm-l] RE: [obm-l] Paulo e suas ... 



Atencao usuarios:

A lista absolutamente nao pode receber mensagens
deste tipo.
Diferencas pessoais devem ser resolvidas pessoalmente.
Por favor, nao enviem mais mensagens deste teor.



--
From: J.Paulo roxer ´til the end [EMAIL PROTECTED]
To: [EMAIL PROTECTED]
Subject: Re: [obm-l] Re: [obm-l] RE: [obm-l] Paulo e suas ... 
Date: Sun, Jul 13, 2003, 5:37 PM


Se reclamar e questionar fosse chatice,os filósofos seriam assassinados.

Em vez de me chamar de chato,por q não sugere algum lugar pra colocar minhas dúvidas?Vc contribuiria mais.

João Paulo
- Original Message - 
From: [EMAIL PROTECTED] mailto:[EMAIL PROTECTED] 
To: [EMAIL PROTECTED] mailto:[EMAIL PROTECTED] 
Sent: Sunday, July 13, 2003 4:12 PM
Subject: [obm-l] Re: [obm-l] RE: [obm-l] Paulo e suas ... 

Vc só consegue me chamar de idiota,porq sou o único leigo.Se a maioria
aqui
não fosse campeão de olimpíadas,teriam a minha reação,pediriam mais explicação
para tantas questões entediantes,que a maioria não entende(Os normais).

.. vc não está vendo que está reclamando no lugar errado ?
deixa de ser chato !!


Mathematicus nascitur, non fit
Matemáticos não são feitos, eles nascem
---
Gabriel Haeser
www.gabas.cjb.net http://www.gabas.cjb.net 


--
Use o melhor sistema de busca da Internet
Radar UOL - http://www.radaruol.com.br



=
Instruções para entrar na lista, sair da lista e usar a lista em
http://www.mat.puc-rio.br/~nicolau/olimp/obm-l.html
=

Email.it, the professional e-mail, gratis per te: clicca qui http://www.email.it/cgi-bin/start?sid=3 

Sponsor:
Un giorno a settimana gusta la Mozzarella di Bufala Campana e la Treccia di Agerola.
Clicca qui http://adv.email.it/cgi-bin/foclick.cgi?mid=982d=13-7 






[obm-l] Re: [obm-l] Re: [obm-l]_Olá!

2003-06-29 Por tôpico Wagner

Mais geralmente ainda se lim(x-a) f(x) = lim(x-a) g(x) = 0,
lim(x-a) (f(x))^(g(x)) = lim(x-a) (g(x))^(f(x)) = 1

André T.


- Original Message -
From: Carlos Gustavo Tamm de Araujo Moreira [EMAIL PROTECTED]
To: [EMAIL PROTECTED]
Sent: Saturday, June 28, 2003 2:36 PM
Subject: Re: [obm-l] Re: [obm-l]_Olá!


Definicao nao se demonstra, mas vou mencionar dois fatos a favor de
 definir 0^0=1:
 i) Pelo binomio de Newton, 0^k=(1-1)^k=Soma(j=0 ate' k)(C(k,j).(-1)^j,
para
 todo k natural. Fazendo k=0, temos 0^0=C(0,0).(-1)^0=0!/(0!.0!)=1 (note
que
 1=1!=1.0! mostra que 0!=1 e' a definicao natural de 0!). Em geral, para
 aplicacoes em combinatoria, parece natural definir 0^0=1.
 ii) lim(x-0)(x^x)=1, e ,mais geralmente, para todo real a  0,
 lim(x-0)(x^(x^a))=1. Assim, em muitos problemas de calculo, ou
analise,como
 queiram (eu arriscaria dizer na maioria) o valor mais natural de 0^0 e' 1.
Abracos,
 Gugu

 
 Nicolau, confesso que não tinha conhecimento desta definição para 0^0.
Fui
 tentar ajudar e acabei atrapalhando. Obrigado pelo esclarecimento.
 Abraços. Fabio.
 
 
 
  Em 27 Jun 2003, [EMAIL PROTECTED] escreveu:
 
 On Fri, Jun 27, 2003 at 04:45:10AM -0300, Fabio Henrique wrote:
  Thiago, desculpe me intrometer. O que você diz é verdade. Por isso,
0/0 é
  INDETERMINADO. Pode-se estender este raciocínio para 0^0. Pense
comigo:
 0^0
  = 0^k/0^k com k diferente de zero. Mas 0^k = 0. Logo, 0^0 = 0/0 que é
  indeterminado.
 
 Esse papo de indeterminado só deve ser usado quando se fala de
limites.
 
 O usual é não definir 0/0 e definir 0^0 = 1.
 
 De novo, isso são definições.
 
 []s, N.

=
 Instruções para entrar na lista, sair da lista e usar a lista em
 http://www.mat.puc-rio.br/~nicolau/olimp/obm-l.html

=
 
 --
 
 _
 Voce quer um iGMail protegido contra vírus e spams?
 Clique aqui: http://www.igmailseguro.ig.com.br
 Ofertas imperdíveis! Link: http://www.americanas.com.br/ig/
 Ofertas imperdíveis!
 
 =
 Instruções para entrar na lista, sair da lista e usar a lista em
 http://www.mat.puc-rio.br/~nicolau/olimp/obm-l.html
 =

 =
 Instruções para entrar na lista, sair da lista e usar a lista em
 http://www.mat.puc-rio.br/~nicolau/olimp/obm-l.html
 =


=
Instruções para entrar na lista, sair da lista e usar a lista em
http://www.mat.puc-rio.br/~nicolau/olimp/obm-l.html
=


[obm-l] Re: [obm-l] Potência

2003-06-29 Por tôpico Wagner
Analise as conguências módulo desse número, isso pode te dar uma dica
de quais devem ser as congruências módulo de b.

André T.

- Original Message -
From: cfgauss77 [EMAIL PROTECTED]
To: Lista OBM [EMAIL PROTECTED]
Sent: Saturday, June 28, 2003 4:26 PM
Subject: [obm-l] Potência


Seja o número a=^, o núemo b obtido da soma de
todos os algarismos do número a e, finalmente, o número
c obtido da soma de todos os algarismos de b. Determine
o número c.

Desde já agradeço


__
Seleção de Softwares UOL.
10 softwares escolhidos pelo UOL para você e sua família.
http://www.uol.com.br/selecao


=
Instruções para entrar na lista, sair da lista e usar a lista em
http://www.mat.puc-rio.br/~nicolau/olimp/obm-l.html
=


=
Instruções para entrar na lista, sair da lista e usar a lista em
http://www.mat.puc-rio.br/~nicolau/olimp/obm-l.html
=


[obm-l] Re: [obm-l] Olimpíadas ao redor do mundo....

2003-06-22 Por tôpico Wagner



Oi para todos!

Sejam x, y e z=180º-(x+y) os 3 ângulos do 
triângulo.
Usando tg(x+y) = (tg(x) + tg(y))/(1 - tg(x)tg(y)) e 
tg(180º-x) = -tg(x),
tg(z) = (tg(x) + tg(y))/(tg(x)tg(y) 
-1)
Então basta resolver a equação 
a = (b+c)/(bc-1) = abc -a =b+c = abc = 
a+b+c.

É fácil ver que (1,2,3) é resposta
Falta provar que essa é a única 
resposta.
Se não me engano isso caiu na Unicamp em 2001 (2ª 
fase)

André T.


  - Original Message - 
  From: 
  [EMAIL PROTECTED] 
  To: [EMAIL PROTECTED] 
  Sent: Sunday, June 22, 2003 6:49 PM
  Subject: [obm-l] Olimpíadas ao redor do 
  mundo
  Estou resolvendo o exercício abaixo, quase todo por 
  inspeção. Calculei tg de 15 graus, tg de 75 graus, arctgb=2, obtendo 
  b=60graus mais um acréscimo x.., arctga=3, obtendo a=75 graus menos y e 
  assim por diante...Ja da para concluir algumas coisas, mas gostaria de saber 
  se existe um caminho menos braçal, ou, se não houver, gostaria que me 
  confirmassem... Desde já 
  agradeço, 
  Crom.Espanha-1998As tangentes dos ãngulos de um triângulo são inteiros 
  positivos. Determine estes números. 


Re: [obm-l] integral

2003-06-22 Por tôpico Wagner
Oi para todos!

e^x = Sum {i=0 ; oo} (x^n/n!) =
e^x/x = 1/x + Sum {i=1 ; oo} (x^n/(n-1)!) =
int. [e^x/x] = ln |x| + Sum{i=1 ; oo} (x^(n+1)/((n+1)(n-1)!))

André T.

- Original Message -
From: adr.scr.m [EMAIL PROTECTED]
To: [EMAIL PROTECTED]
Sent: Sunday, June 22, 2003 10:59 PM
Subject: [obm-l] integral


alguém poderia me ajudar na integral que me deram :
integral [(e^x) / x ] dx.

obrigado.


__
Seleção de Softwares UOL.
10 softwares escolhidos pelo UOL para você e sua família.
http://www.uol.com.br/selecao


=
Instruções para entrar na lista, sair da lista e usar a lista em
http://www.mat.puc-rio.br/~nicolau/olimp/obm-l.html
=


=
Instruções para entrar na lista, sair da lista e usar a lista em
http://www.mat.puc-rio.br/~nicolau/olimp/obm-l.html
=


[obm-l] (tg x)/x

2003-06-14 Por tôpico Wagner



Oi para todos!

A integral abaixo é elementar?

 /\
|
| (tg x)/x 
dx
 |
\/

Tentei integrar ela mas não consegui chegar a lugar 
nenhum.

André T.


[obm-l] Séries

2003-06-05 Por tôpico Wagner



Oi para todos!

 Como resolver essa:

Seja S(x) uma série de potências tal que e^S(x) = 
sen x.
Escreva S(x) em sua forma compacta.

André T.


Re: [obm-l] Quem sabe?

2003-06-04 Por tôpico Wagner
Ninguém sabe



- Original Message -
From: André W.Hirano [EMAIL PROTECTED]
To: [EMAIL PROTECTED]
Sent: Tuesday, June 03, 2003 11:12 AM
Subject: [obm-l] Quem sabe?


 P=NP?








 
 Obtenha seu Serviço de Correio eletrônico Baseado na  Web Service em
http://www.zzn.com
 =
 Instruções para entrar na lista, sair da lista e usar a lista em
 http://www.mat.puc-rio.br/~nicolau/olimp/obm-l.html
 =


=
Instruções para entrar na lista, sair da lista e usar a lista em
http://www.mat.puc-rio.br/~nicolau/olimp/obm-l.html
=


Re: [obm-l] Duvidas

2003-06-04 Por tôpico Eduardo Wagner
Porismo nao consta dos nossos dicionarios.
Porismo vem do frances porisme que significa
uma afirmacao muito facil de demonstrar. Pode ser um lema
ou um corolario, algo que nao tem o status de teorema.

Abracos,

Wagner.

--
From: Nicolau C. Saldanha [EMAIL PROTECTED]
To: [EMAIL PROTECTED]
Subject: Re: [obm-l] Duvidas
Date: Tue, Jun 3, 2003, 4:52 PM


 On Tue, Jun 03, 2003 at 07:44:31PM +, Antonio Neto wrote:
E haveria alguem da minha prisca geracao que lembrasse o que eh um
 escohlio? Abracos, olavo.

 Bem lembrado, um escólio é algo que segue facilmente não do *enunciado*
 de um teorema (como um corolário) mas da *demonstração* do teorema.

 E porismo? Eu conheço o porismo de Poncelet.

 []s, N.
 =
 Instruções para entrar na lista, sair da lista e usar a lista em
 http://www.mat.puc-rio.br/~nicolau/olimp/obm-l.html
 =
=
Instruções para entrar na lista, sair da lista e usar a lista em
http://www.mat.puc-rio.br/~nicolau/olimp/obm-l.html
=


Re: [obm-l] integral

2003-06-04 Por tôpico Wagner
Oi para todos!

Encontrei uma solução usando séries infinitas:
Sugiro acompanhar escrevendo pois a notação usada pode ficar confusa

Primeiro fazemos a mudança de variável u=x+1.
Então dx = du.
Então G(x)=int. ((sen x)/(x+1))dx = int. ((sen (u-1))/u)du = F(u)
Usando a série de Taylor de sen x:

F(u) =int. [SUM  ((u-1)^(2i+1))((-1)^i)/(2i+1)!(u)]du
  i=0
Usando o binômio de Newton:
2i+1
F(u) = int.[SUM [SUM ((u)^j)((-1)^(2i-j+1))((-1)^i)(binominal( j ,
2i+1))]/(2i+1)!(u)]du
  i=0 j=0
Abrindo o binominal:
2i+1
F(u) = int.[SUM [SUM ((u)^j)((-1)^(3i-j+1))(2i+1)!/(2i-j+1)!(2i+1)!(
j )!(u)]]du
  i=0 j=0
Simplificando:
   2i+1
F(u)= int.[SUM [SUM ((u)^j)((-1)^(3i-j+1))/(2i-j+1)!( j )!(u)]]du =
 i=0 j=0
  2i+1
int. [SUM [SUM ((u)^(j-1))((-1)^(3i-j+1))/(2i-j+1)!( j )!]  + SUM
[(-1)^(3i+1)/(u)(2i+1)!]]du
 i=0 j=1
i=0
Agora calculamos a integral:
   2i+1
F(u) = [SUM [SUM ((u)^j)((-1)^(3i-j+1))/( j )( j )!(2i-j+1)!]   + SUM [ (ln
(u))((-1)^(3i+1))/(2i+1)!]
 i=0  j=1
i=0
Desfazendo a mudança de variável chegamos ao resultado:
  2i+1
int. [(sen(x))/(x+1)]dx = SUM [ SUM ((x+1)^j)((-1)^(3i-j+1))/( j )(
j )!(2i-j+1)!]  + SUM [ (ln(x+1))((-1)^(3i+1))/(2i+1)!]
   i=0  j=1
i=0

André T.



- Original Message -
From: carlos augusto [EMAIL PROTECTED]
To: [EMAIL PROTECTED]
Cc: [EMAIL PROTECTED]
Sent: Monday, June 02, 2003 4:04 PM
Subject: [obm-l] integral


 Sou aluno do 1º período do curso de ciência da
 computação, e não consegui responder a seguinte
 questão.

 Resolver a integral:
   /
   | Sen(x)
   | -- dx
   | 1 + x
   /
 resposta: Sen(x - Log(1 + x)) by Mathematica.


 ___
 Yahoo! Mail
 Mais espaço, mais segurança e gratuito: caixa postal de 6MB, antivírus,
proteção contra spam.
 http://br.mail.yahoo.com/
 =
 Instruções para entrar na lista, sair da lista e usar a lista em
 http://www.mat.puc-rio.br/~nicolau/olimp/obm-l.html
 =



=
Instruções para entrar na lista, sair da lista e usar a lista em
http://www.mat.puc-rio.br/~nicolau/olimp/obm-l.html
=


[obm-l] Matriz de Hilbert

2003-05-29 Por tôpico Wagner



Quais são as aplicações da matriz de 
Hilbert?

Só para lembrar, a matriz de Hilbert Hn é a matriz de termos
da forma aij = 
1/(i+j-1) e ordem n.

André T.


[obm-l] Re: [obm-l] Demonstrações

2003-04-05 Por tôpico Wagner
Title: Mensagem



Oi para todos!

TEOREMA: Se a é umnº naturalque não é 
um quadrado perfeito, sqrt(a) é irracional
PROVA: Suponha por absurdo que sqrt(a) é racional. 
Logo sqrt(a) pode ser escrito na forma p/q , mdc(p,q)=1
Logo existe solução racionalpara p e q tais 
que mdc(p,q)=1para a=p^2/q^2 = a.q^2 = p^2 . a,p,q são 
inteiros.
Logop é divisível por a. Logo p = a.r para 
algum valor inteiro de r . Logo a^2.r^2 = a.q^2 = q^2 = a.r^2 . a,q,r 
são
inteiros .Segue que q é divisível por a. Como a não 
é quadrado perfeito, a1 . Logo mdc(p,q)1 . Absurdo !
PROPRIEDADE: Se a é irracional, sqrt(a) também é 
irracional

Usandoesses teoremas acima fica fácil provar 
os 2 primeiros

1)Eleve ( sqrt(3) + sqrt(5)) ao quadrado, você terá 
8 + 2sqrt(15) que é irracional pois sqrt(15) é irracional, uma vez 
que
15 não é quadrado perfeito, logo sqrt(8 + 
2sqrt(15)) = sqrt(3) + sqrt(5) é irracional.
2)(sqrt(p) + sqrt(q))^2 = p+q + 2sqrt(p.q) . Como p 
e q são primos distintos p.q não é quadrado perfeito, logo sqrt(p) + 
sqrt(q)é irracional

André T.



  - Original Message - 
  From: 
  Hely Jr. 
  
  To: [EMAIL PROTECTED] 
  Sent: Thursday, April 03, 2003 10:58 
  PM
  Subject: [obm-l] Demonstrações
  
  Alguem poderia me 
  ajudar nestas demonstrações
  
  1) sabendo que 
  sqrt(3) e sqrt(5) são irracionais, verifique que sqrt(3) + sqrt(5) é 
  irracional.
  
  2) sejam p 0 e 
  q0 primos distintos. verifique que sqrt(p) + sqrt(q) é 
  irracional
  
  3) se p e q sào 
  inteiros positivos distintos e pelo menos um dos numeros sqrt(p) ou sqrt(q) é 
  irracional, então sqrt(p) + sqrt(q) é tb irracional.
  
  desde ja 
  agradeço


Re: [obm-l] geometria plana

2003-04-05 Por tôpico Wagner



Oi para todos!

Seja ABCD esse losango. Seja E o encontro das 
diagonais AC e BD. Logo E é o centro da circunferência.
Tome o triângulo retângulo ABE de catetos 9 cm e 12 
cm . Logo AB^2 = 81 + 144 = 225 = AB = 15 cm.
O raior da circunferência é igual a altura de 
ABE em relação a base AB. Sejax a área do triângulo ABE.
Então x = 12.9/2 = 15.r/2 = r = 7,2 cm = 
a=51,84pi.
Genericamente se as diagonais medissem d e D, 
usando uma resolução análoga teriamos a=[(d^2.D^2)/4(d^2+D^2)].pi

André T.




  - Original Message - 
  From: 
  [EMAIL PROTECTED] 
  
  To: [EMAIL PROTECTED] 
  Sent: Saturday, April 05, 2003 7:31 
  PM
  Subject: [obm-l] geometria plana
  Olá pessoal, As 
  diagonais de um losango medem 18 cm e 24 cm. Qual é a área do círculo inscrito 
  neste losango? Obs: Eu não consigo achar o raio da circunferência 
  inscrita, fazendo uma figura dá para perceber que o raio é um pouco menor que 
  9, já que o lado menor mede 18. Mas procuro um relação entre estas duas 
  regiões planas, pois não temos aqui uma ciircunferência inscrita em uma 
  polígono regular. 


[obm-l] sqrt(12a^3 - 3)

2003-03-31 Por tôpico Wagner



Oi para todos!

Deêm uma olhada nesse problema abaixo:

Prove que sea é racional, então sqrt(12a^3 - 3) só é racional sea = 1

André T.


Re: [obm-l] Mais Probls em Aberto II

2003-03-31 Por tôpico Wagner
Oi para todos!

Isso tambm  a prova das 2 hipteses que eu sugeri para resolver o problema
(Mas essas hipteses no eram suficientes para chegar na resposta, j que a
resposta poderia ser 3^2000 ou 3^2001)

Andr T.



  17)
  a) Ao escrevermos a frao 1/3^2002 como um nmero decimal, obtemos uma
  dzima peridica. Qual o nmero de algarismos da perodo? b) Existe
algum
  inteiro positivo n tal que 1/3^n  uma dzima peridica cujo perodo tem
um
  nmero par de algarismos?
  [...]

 17)
 a)
 O conjunto formados pelos invertveis mdulo 3^2002 que so congruentes a
1
 mdulo 9  invariante por uma multiplicao por 10. Esse conjunto tem
 \phi(3^2002)/\phi(9) = 3^2000 elementos. Seja P o produto de todos os seus
 elementos. Ento

 P === 10^(3^2000)*P (mod 3^2002)
 10^(3^2000) === 1 (mod 3^2002)

 Logo a ordem de 10 (mod 3^2002) divide 3^2000. Como a ordem de 10 (mod
3^2002)
  o nmero de elementos do menor conjunto invariante por uma multiplicao
 por 10, mas como 10 !== 1 (mod 27), no existem conjuntos com
 \phi(3^2002)/\phi(27) === 3^1999 elementos. Logo a ordem de 10 (mod
3^2002) 
 mesmo 3^2000, *logo o perodo de 1/3^2002  3^2000*.

 b)
 No. Seja K = {x | x  invertvel (mod 3^n) e x === 1 (mod 9)}.  bvio
que
 10K = K. Se P  o produto dos elementos de K, ento

 P === 10^(#(K))*P (mod 3^n)
 10^(#(K)) === 1 (mod 3^n)
 ord_3^2002(10) | #(K) = \phi(3^n)/\phi(9) = 3^(n-2)

 Mas 2 | ord_3^2002(10) = 2 | 3^(n-2), *absurdo*. Logo 1/3^n sempre tem
 perodo mpar (em particular, seu perodo sempre  uma potncia de trs).

 []s,

 - --
 Fbio ctg \pi Dias Moreira
 -BEGIN PGP SIGNATURE-
 Version: GnuPG v1.0.6 (GNU/Linux)
 Comment: For info see http://www.gnupg.org

 iD8DBQE+iNnmalOQFrvzGQoRAopiAKCnIycHoC8alkkUs3Rs40pYdFi3oACcDmo+
 aegviRKBOA7fJIQz24jyDWk=
 =+m/H
 -END PGP SIGNATURE-

 =
 Instrues para entrar na lista, sair da lista e usar a lista em
 http://www.mat.puc-rio.br/~nicolau/olimp/obm-l.html
 O administrador desta lista  [EMAIL PROTECTED]
 =


=
Instrues para entrar na lista, sair da lista e usar a lista em
http://www.mat.puc-rio.br/~nicolau/olimp/obm-l.html
O administrador desta lista  [EMAIL PROTECTED]
=


Re: [obm-l] Limites

2003-03-29 Por tôpico Wagner
Oi para todos!

lim(x-1)  (x^2-x)/(2x^2+5x-7)=
lim(x-1)  (x-1).(x)/2(x-1)(x+7/2)=
lim(x-1)  x/(2x+7) = 1/9

lim(x-5)  (3x^2-13x-10)/(2x^2-7x-15)=
lim(x-5)  3(x-5)(x+2/3)/2(x-5)(x+3/2)=
lim(x-5)  (3x+2)/(2x+3) = 17/13

coeficiente angular da tangente de f(a) = f '(a)
Pela regra g(x)=b.x^n = g '(x)=b.n.x^(n-1)
f '(x) = 5.2.x^(2-1) - 4.1.x^(1-1) = 10x - 4=
f '(a) = 10a - 4
Ou pela definição:
f '(x) = lim(h-0) (f(x+h)-f(x))/h =
lim(h-0)  (5(x+h)^2 -4(x+h) - (5x^2 - 4x))/h =
lim(h-0)  (5x^2 + 10xh + 5h^2 -4x -4h - 5x^2 + 4x)/h =
lim(h-0)  h.(10x - 4 + 5h)/h =
lim(h-0)  10x - 4 + 5h = 10x - 4 = f '(x) =
f '(a) = 10a - 4


André T.


- Original Message -
From: [EMAIL PROTECTED]
To: [EMAIL PROTECTED]
Sent: Saturday, March 29, 2003 4:10 PM
Subject: [obm-l] Limites


 Olá,

 Gostaria de ver a resolucao desses exercicios:

 Determinar os limites:

 lim(x-1) (x^2 - x)/(2x^2 + 5x - 7)
 Resposta: 1/9

 lim(x-5) (3x^2 - 13x - 10)/(2x^2 - 7x - 15)
 Resposta: 17/13


 Determinar o coeficiente angular da tangente ao grafico de f no ponto P(a,
f(a)):


 f(x) = 5x^2 - 4x
 Resposta: 10a - 4


 Esses exercicios sao do livro do Swokowski.



 Agradeço quem ajudar,

 Gabriel Campos Pérgola





 http://www.ieg.com.br
 =
 Instruções para entrar na lista, sair da lista e usar a lista em
 http://www.mat.puc-rio.br/~nicolau/olimp/obm-l.html
 O administrador desta lista é [EMAIL PROTECTED]
 =



=
Instruções para entrar na lista, sair da lista e usar a lista em
http://www.mat.puc-rio.br/~nicolau/olimp/obm-l.html
O administrador desta lista é [EMAIL PROTECTED]
=


[obm-l] Fermat

2003-03-24 Por tôpico Wagner



Oi para todos!

 Aonde posso encontrar na 
internet sobre os teoremas elaborados por Fermat?

André T.


[obm-l] Re: [obm-l] Determine o nº de algarismos do período.

2003-03-23 Por tôpico Wagner



Oi para todos!

Vamos tentar encontrar um padrão para o nº de 
algarismos do período de
1/3^k para k natural maior do que 1:

Para1/3 , o nº de algarismos do período é 1 
(0,333...)
Para 1/3^2 , esse nº também é 1 
(0,111...)
Para 1/3^3 , esse nº é 3 (0,037037...)
Para 1/3^4 , esse nº é 9 
(0,012345679...)
Para 1/3^5 , esse nº é 27 
(0,004115226337448559670781893...)

SUGESTÕES: 
-Tente provar que o nº de algarismos dos 
períodosserá sempre 
da forma 3^a,com a natural maior que 1.
-Tente provar que se k2, o nº de algarismos do 
período de1/3^(k+1) 
é maior (ou então maior ou igual) que o nº de 
algarismos de 1/3^k.

OBS:
Eu não tentei provar nenhuma das 2 afirmações, logo 
elas podem ser falsas.
Mas, é fácil perceber que se ambas estiverem 
corretas a resposta é 3^2001.


André T.



  - Original Message - 
  From: 
  André Riker 
  To: [EMAIL PROTECTED] 
  Sent: Friday, March 21, 2003 10:30 
  PM
  Subject: [obm-l] Determine o nº de 
  algarismos do período.
  
  
  Alguém poderia me ajudar a resulver esse problema?
  Ao escrevermos a fração 1/3²ºº² como um número decimal, obtemos 
  uma dízima periódica. Qual o número de algarismos da período?
  Obrigado, André
   
   
  
  
  
  Busca Yahoo! O serviço de 
  busca mais completo da Internet. O que você pensar o Yahoo! 
encontra.


[obm-l] Fw: [obm-l] Determine o nº de algarismos do período.

2003-03-23 Por tôpico Wagner




Oi para todos!

Só para corrigir a mensagem anterior, se as duas 
afirmações estiverem
corretas a respostapode ser tanto3^2000 
como 3^2001.
Isso segue do teorema de que o nº de algarismos do 
período de 1/x
é menordo quex, para todo x natural 
maior que 1 (o período de 1 têm 1
algarismo: 1,... ).
Esse teorema é interessante por que prova que um 
número decimal com
período infinito (ou sem período definido) é 
obrigatóriamente irracional.
A prova dele é fácil e se alguém quiser eu coloco 
na lista.


Desculpem a distração
André T.



  - Original Message - 
  From: 
  Wagner 
  To: [EMAIL PROTECTED] 
  Sent: Sunday, March 23, 2003 12:40 
  PM
  Subject: Re: [obm-l] Determine o nº de 
  algarismos do período.
  
  Oi para todos!
  
  Vamos tentar encontrar um padrão para o nº de 
  algarismos do período de
  1/3^k para k natural maior do que 1:
  
  Para1/3 , o nº de algarismos do período é 1 
  (0,333...)
  Para 1/3^2 , esse nº também é 1 
  (0,111...)
  Para 1/3^3 , esse nº é 3 
  (0,037037...)
  Para 1/3^4 , esse nº é 9 
  (0,012345679...)
  Para 1/3^5 , esse nº é 27 
  (0,004115226337448559670781893...)
  
  SUGESTÕES: 
  -Tente provar que o nº de algarismos dos 
  períodosserá sempre 
  da forma 3^a,com a natural maior que 1.
  -Tente provar que se k2, o nº de algarismos 
  do período de1/3^(k+1) 
  é maior (ou então maior ou igual) que o nº de 
  algarismos de 1/3^k.
  
  OBS:
  Eu não tentei provar nenhuma das 2 afirmações, 
  logo elas podem ser falsas.
  Mas, é fácil perceber que se ambas estiverem 
  corretas a resposta é 3^2001.
  
  
  André T.
  
  
  
- Original Message - 
From: 
André Riker 
To: [EMAIL PROTECTED] 
Sent: Friday, March 21, 2003 10:30 
PM
Subject: [obm-l] Determine o nº de 
algarismos do período.


Alguém poderia me ajudar a resulver esse problema?
Ao escrevermos a fração 1/3²ºº² como um número decimal, 
obtemos uma dízima periódica. Qual o número de algarismos da período?
Obrigado, André
 
 



Busca Yahoo! O serviço 
de busca mais completo da Internet. O que você pensar o Yahoo! 
  encontra.


[obm-l] Re: [obm-l] raciocínio lógico

2003-03-17 Por tôpico Wagner



Oi para todos!


 Se a) estivesse certa então o 
conjunto dos metaleiros e o dos preguiçososseria o 
mesmo, o 
que não pode ser afirmado com 
certeza.
 A alternativa b) é 
contraditória, pois segundo 3, b) implicaria que algum estudante é
preguiçoso, o que é uma contradição da sentença 
1.
 Pelo mesmo motivo c) é falsa, 
pois isso implicaria que alguns preguiçosos são 
estudantes, o que contradiz 1.
 A alternativa d) é falsa, pois 
segundo 2, Rodrigo é metaleiro e consequentemente 
preguiçoso segundo 3. Novamente isso implicaria na 
contradição de 1.

 Logo nenhuma alternativa é 
verdadeira, provavelmente deve ter havido um erro de
impressão, pois se d) fosse : Rodrigo não é 
estudante, d) seria verdadeira.
 De qualquer jeito é um 
preconceito contra os metaleiros.


André T.



  - Original Message - 
  From: 
  [EMAIL PROTECTED] 
  
  To: [EMAIL PROTECTED] 
  Sent: Saturday, March 15, 2003 8:32 
  PM
  Subject: [obm-l] raciocínio lógico 
  Olá pessoal, 
  Considere as seguintes sentenças: 1. Nenhum estudante é 
  preguiçoso 2. Rodrigo é metaleiro. 3. Todos os metaleiros são 
  preguiçosos Supondo que as três sentenças são verdadeiras, verifique 
  qual das sentenças a seguir é certamente verdadeira: a) Todos os 
  preguiçosos são metaleiros b) Algum estudante é metaleiro c) Alguns 
  metaleiros são estudantes d) Rodrigo é estudante resposta: "d" 
  Obs: Eu acredito que o gabarito esteja errado, pois seja M contido em 
  P e E disjunto de P não há como quaisquer das alternativas serem verdadeiras. 
  E a alternativa "d" por dois motivos: o diagrama de Venn-Euler e a afirmação 2 
  do enunciado. Observando que esta é uma questão de um livro que muitos aqui 
  têm.  


[obm-l] Potências

2003-03-17 Por tôpico Wagner



Oi pessoal !

 Se a^(b^c) = b^d , c/d pode ser 
dado em função de a e b ?

André T.


Re: [obm-l] (O (sqrt n))

2003-03-07 Por tôpico Wagner



Oi para todos !

Isso implicaria que se f(x) = g(x) + O(h(x)), 
então
lim x --- inf. f(x) = lim x --- inf. 
(g(x) + h(x)) ?


André T.



  - Original Message - 
  From: 
  Cláudio (Prática) 
  To: [EMAIL PROTECTED] 
  Sent: Thursday, March 06, 2003 11:32 
  AM
  Subject: Re: [obm-l] (O (sqrt n))
  
  O(sqrt(n)) representa uma função F, cujo domínio 
  normalmenteé o conjunto dos naturais ou dos reais,tal que | F(n) 
  |= C*sqrt(n), para todo n suficientemente grande, onde C é uma 
  constante que independe de n.
  
  Essa notação (chamada em inglês de "Big-Oh 
  notation") é muito utilizada em teoria dos números e em computação, para 
  representar a ordem de magnitude de uma função ou série cuja soma não se 
  conhece exatamente.
  
  
- Original Message - 
From: 
Wagner 
To: [EMAIL PROTECTED] 
Sent: Wednesday, March 05, 2003 4:34 
PM
Subject: [obm-l] (O (sqrt n))

Oi para todos !

Estava vendo a sequência A006218 no http://www.research.att.com/~njas/sequences/
e me deparei com O(sqrt(n)) na fórmula da 
sequência
Se alguém puder me esclarecer o que isso quer 
dizer 
eu agradeceria muito.

André 
T.


[obm-l] Re: [obm-l] Re:_[obm-l]_Re:_[obm-l]_Re:_[obm-l]_verificações

2003-03-07 Por tôpico Wagner
Oi para todos!

Como você disse, no enunciado consta R^2 e não V^2 , isso
quer dizer que (a,b) não é necessariamente um vetor, por
exemplo (a,b) pode ser um par ordenado, ou um ponto em
um plano, em que a e b são reais.
Note que * é apenas uma operação binária, então mesmo que
(a,b) fosse um vetor, isso não seria uma condição suficiente
para que * apresentasse as propriedades comutativa e
associativa. O que é importante perceber é que essas propriedades
dependem apenas do modo como * se relaciona com os elementos
da forma (a,b) , independentemente da natureza desses elementos.

André T.


- Original Message -
From: pichurin [EMAIL PROTECTED]
To: [EMAIL PROTECTED]
Sent: Friday, March 07, 2003 1:05 PM
Subject: Re: [obm-l] Re:_[obm-l]_Re:_[obm-l]_Re:_[obm-l]_verificações


 OK, mas no enunciado do problema consta R^2.
 Então (a,b) é um vetor.Ou estou errado?Caso (a,b) não
 seja um vetor, ele é o que?

  --- Wagner [EMAIL PROTECTED] escreveu:  Oi pessoal!
 
  Quem disse que (a,b) e (c,d) são vetores e quem
  disse que * é adição ou multiplicação?
 
  André T.
 
 
  - Original Message -
  From: pichurin [EMAIL PROTECTED]
  To: [EMAIL PROTECTED]
  Sent: Sunday, March 02, 2003 2:38 PM
  Subject: Re: [obm-l] Re:
  [obm-l]_Re:_[obm-l]_verificações
 
 
   Desculpe, mas estou aprendendo espaços vetoriais
  agora
   e estou meio perdido nesse assunto.
  
   (a,b)*(c,d)=(c,d)(a,b)
   Isso não é comutativa de adição
   comutativa de adição seria:
   (a,b) + (c,d) = (c,d) + (a,b)
  
  
  
  
--- Nicolau C. Saldanha
   [EMAIL PROTECTED] escreveu:  On
  Sun,
   Mar 02, 2003 at 01:26:40PM -0300, pichurin
wrote:
 Mas as propriedades associativa e comutativa
  valem
 para adições, e não para  a operação *.
 Não entendi.
   
O que o Wagner Timpa observa é que não há nada
  que
garanta
que a+d seja igual a b+c ou que b+c seja igual a
a+d,
que é o que a comutatividade exigiria
  (analogamente
para associatividade).
Está certo, mas acho que com um contraexemplo
  (como
o do Morgado)
fica mais claro.
   
[]s, N.
   
   
  --- Wagner [EMAIL PROTECTED] escreveu:  Oi
  para
 todos!
 
  Se vale a propriedade comutativa:
  (a,b)*(c,d) = (c,d)*(a,b) =
  (a+d , b+c) = (b+c , a+d) . FALSO
 
  Se vale a propriedade associativa:
  [(a,b)*(c,d)]*(e,f) = (a,b)*[(c,d)*(e,f)] =
  (a+d , b+c)*(e,f) = (a,b)*(c+f , d+e) =
  (a+d+f , b+c+e) = (a+d+e , b+c+f) . FALSO
 
  André T.
 
 
  - Original Message -
  From: pichurin [EMAIL PROTECTED]
  To: [EMAIL PROTECTED]
  Sent: Saturday, March 01, 2003 4:33 PM
  Subject: [obm-l] verificações
 
 
   Mostre que no R^2 , a operação
  (a,b)*(c,d)=(a+d,b+c)
   não verifica as propriedades comutativa e
  associativa
   da adição.
   
  
 
 =
Instruções para entrar na lista, sair da lista e
usar a lista em
   
  http://www.mat.puc-rio.br/~nicolau/olimp/obm-l.html
O administrador desta lista é
[EMAIL PROTECTED]
   
  
 
 =
  
  
 
 ___
   Busca Yahoo!
   O serviço de busca mais completo da Internet. O
  que você pensar o Yahoo!
  encontra.
   http://br.busca.yahoo.com/
  
 
 =
   Instruções para entrar na lista, sair da lista e
  usar a lista em
  
  http://www.mat.puc-rio.br/~nicolau/olimp/obm-l.html
   O administrador desta lista é
  [EMAIL PROTECTED]
  
 
 =
 
 
 
 =
  Instruções para entrar na lista, sair da lista e
  usar a lista em
  http://www.mat.puc-rio.br/~nicolau/olimp/obm-l.html
  O administrador desta lista é
  [EMAIL PROTECTED]
 
 =

 ___
 Busca Yahoo!
 O serviço de busca mais completo da Internet. O que você pensar o Yahoo!
encontra.
 http://br.busca.yahoo.com/
 =
 Instruções para entrar na lista, sair da lista e usar a lista em
 http://www.mat.puc-rio.br/~nicolau/olimp/obm-l.html
 O administrador desta lista é [EMAIL PROTECTED]
 =


=
Instruções para entrar na lista, sair da lista e usar a lista em
http://www.mat.puc-rio.br/~nicolau/olimp/obm-l.html
O administrador desta lista é [EMAIL PROTECTED]
=


[obm-l] (O (sqrt n))

2003-03-05 Por tôpico Wagner



Oi para todos !

Estava vendo a sequência A006218 no http://www.research.att.com/~njas/sequences/
e me deparei com O(sqrt(n)) na fórmula da 
sequência
Se alguém puder me esclarecer o que isso quer dizer 

eu agradeceria muito.

André T.


Re: [obm-l] Problemas em Aberto III

2003-03-03 Por tôpico Wagner
Oi pessoal !

Parece que falta alguma coisa no enunciado do problema 25, do modo
como o problema é apresentado V  10U não garante que a nave vá encontrar o
alienígena. Por exemplo, o alienígena e a nave podem ficar andando em
trajetórias
circulares que não se cruzam.

André T.

- Original Message -
From: Nicolau C. Saldanha [EMAIL PROTECTED]
To: [EMAIL PROTECTED]
Sent: Sunday, March 02, 2003 10:04 AM
Subject: Re: [obm-l] Problemas em Aberto III


 On Thu, Feb 27, 2003 at 03:04:48PM -0300, Cláudio (Prática) wrote:
  24) Prove que a soma dos comprimentos dos lados de um poliedro
  convexo qualquer é maior que 3 vezes a maior distancia entre dois
vertices
  do poliedro.

 Sejam x e y vértices a distância máxima. Queremos construir três
 caminhos formados por arestas indo de x até y, e estes caminhos
 devem ser disjuntos (exceto por x, y e possivelmente algum vértice
 no meio). Ora, por maxflow-mincut estes três caminhos só *não*
 existem se existir um corte feito por duas arestas, ou seja,
 se existirem duas arestas que, se removidas, desconectam o grafo
 formado por vértices e arestas. Mas isso estaria em contradição
 com o fato de termos um poliedro convexo.

  25) Um alienígena move-se na superfície de um planeta com velocidade
  não superior a U. Uma espaçonave que procura pelo alienígena move-se com
  velocidade V. Prove que a espaçonave sempre  poderá encontrar o
alinígena
  se V  10U.

 Não entendi nada. Quando é que a nave encontra o alienígena?

 []s, N.
 =
 Instruções para entrar na lista, sair da lista e usar a lista em
 http://www.mat.puc-rio.br/~nicolau/olimp/obm-l.html
 O administrador desta lista é [EMAIL PROTECTED]
 =



=
Instruções para entrar na lista, sair da lista e usar a lista em
http://www.mat.puc-rio.br/~nicolau/olimp/obm-l.html
O administrador desta lista é [EMAIL PROTECTED]
=


[obm-l] Re: [obm-l] Re: [obm-l]_Re:_[obm-l]_verificações

2003-03-03 Por tôpico Wagner
Oi pessoal!

Quem disse que (a,b) e (c,d) são vetores e quem
disse que * é adição ou multiplicação?

André T.


- Original Message -
From: pichurin [EMAIL PROTECTED]
To: [EMAIL PROTECTED]
Sent: Sunday, March 02, 2003 2:38 PM
Subject: Re: [obm-l] Re: [obm-l]_Re:_[obm-l]_verificações


 Desculpe, mas estou aprendendo espaços vetoriais agora
 e estou meio perdido nesse assunto.

 (a,b)*(c,d)=(c,d)(a,b)
 Isso não é comutativa de adição
 comutativa de adição seria:
 (a,b) + (c,d) = (c,d) + (a,b)




  --- Nicolau C. Saldanha
 [EMAIL PROTECTED] escreveu:  On Sun,
 Mar 02, 2003 at 01:26:40PM -0300, pichurin
  wrote:
   Mas as propriedades associativa e comutativa valem
   para adições, e não para  a operação *.
   Não entendi.
 
  O que o Wagner Timpa observa é que não há nada que
  garanta
  que a+d seja igual a b+c ou que b+c seja igual a
  a+d,
  que é o que a comutatividade exigiria (analogamente
  para associatividade).
  Está certo, mas acho que com um contraexemplo (como
  o do Morgado)
  fica mais claro.
 
  []s, N.
 
 
--- Wagner [EMAIL PROTECTED] escreveu:  Oi para
   todos!
   
Se vale a propriedade comutativa:
(a,b)*(c,d) = (c,d)*(a,b) =
(a+d , b+c) = (b+c , a+d) . FALSO
   
Se vale a propriedade associativa:
[(a,b)*(c,d)]*(e,f) = (a,b)*[(c,d)*(e,f)] =
(a+d , b+c)*(e,f) = (a,b)*(c+f , d+e) =
(a+d+f , b+c+e) = (a+d+e , b+c+f) . FALSO
   
André T.
   
   
- Original Message -
From: pichurin [EMAIL PROTECTED]
To: [EMAIL PROTECTED]
Sent: Saturday, March 01, 2003 4:33 PM
Subject: [obm-l] verificações
   
   
 Mostre que no R^2 , a operação
(a,b)*(c,d)=(a+d,b+c)
 não verifica as propriedades comutativa e
associativa
 da adição.
 
 =
  Instruções para entrar na lista, sair da lista e
  usar a lista em
  http://www.mat.puc-rio.br/~nicolau/olimp/obm-l.html
  O administrador desta lista é
  [EMAIL PROTECTED]
 
 =

 ___
 Busca Yahoo!
 O serviço de busca mais completo da Internet. O que você pensar o Yahoo!
encontra.
 http://br.busca.yahoo.com/
 =
 Instruções para entrar na lista, sair da lista e usar a lista em
 http://www.mat.puc-rio.br/~nicolau/olimp/obm-l.html
 O administrador desta lista é [EMAIL PROTECTED]
 =


=
Instruções para entrar na lista, sair da lista e usar a lista em
http://www.mat.puc-rio.br/~nicolau/olimp/obm-l.html
O administrador desta lista é [EMAIL PROTECTED]
=


[obm-l] Re: [obm-l] verificações

2003-03-01 Por tôpico Wagner
Oi para todos!

Se vale a propriedade comutativa:
(a,b)*(c,d) = (c,d)*(a,b) =
(a+d , b+c) = (b+c , a+d) . FALSO

Se vale a propriedade associativa:
[(a,b)*(c,d)]*(e,f) = (a,b)*[(c,d)*(e,f)] =
(a+d , b+c)*(e,f) = (a,b)*(c+f , d+e) =
(a+d+f , b+c+e) = (a+d+e , b+c+f) . FALSO

André T.


- Original Message -
From: pichurin [EMAIL PROTECTED]
To: [EMAIL PROTECTED]
Sent: Saturday, March 01, 2003 4:33 PM
Subject: [obm-l] verificações


 Mostre que no R^2 , a operação (a,b)*(c,d)=(a+d,b+c)
 não verifica as propriedades comutativa e associativa
 da adição.

 ___
 Busca Yahoo!
 O serviço de busca mais completo da Internet. O que você pensar o Yahoo!
encontra.
 http://br.busca.yahoo.com/
 =
 Instruções para entrar na lista, sair da lista e usar a lista em
 http://www.mat.puc-rio.br/~nicolau/olimp/obm-l.html
 O administrador desta lista é [EMAIL PROTECTED]
 =


=
Instruções para entrar na lista, sair da lista e usar a lista em
http://www.mat.puc-rio.br/~nicolau/olimp/obm-l.html
O administrador desta lista é [EMAIL PROTECTED]
=


[obm-l] Re: [obm-l] subespaços vetoriais

2003-03-01 Por tôpico Wagner
Oi pessoal.

O espaço vetorial também deve obedecer essas propriedades

André T.

- Original Message -
From: pichurin [EMAIL PROTECTED]
To: [EMAIL PROTECTED]
Sent: Saturday, March 01, 2003 4:26 PM
Subject: [obm-l] subespaços vetoriais


 Diz a teoria de álgebra que se V é um espaço vetorial,
 V é subespaço vetorial de V.
 Tome o Espaço vetorial V={-3,-2,-1,0,1,2,3}
 Se V é espaço vetorial de V, ele deve obedecer as três
 propriedades fundamentais de subespaços:
 -Deve conter o vetor nulo de V
 -se u,v E V, u + v E V, sendo que u e v são vetores de
 V
 -se u E V, au E V, sendo que a é um número real.

 Observe a segunda propriedade:
 -se u,v E V, u + v E V, sendo que u e v são vetores de
 V
 Tomemos como exempo os inteiros 2 e 3 de V
 2 ,3 E V, mas 2+3=5  e 5 não pertence a V
 Então, V não pode ser subespaço de V
 Onde ocorreu o equívoco?

 ___
 Busca Yahoo!
 O serviço de busca mais completo da Internet. O que você pensar o Yahoo!
encontra.
 http://br.busca.yahoo.com/
 =
 Instruções para entrar na lista, sair da lista e usar a lista em
 http://www.mat.puc-rio.br/~nicolau/olimp/obm-l.html
 O administrador desta lista é [EMAIL PROTECTED]
 =


=
Instruções para entrar na lista, sair da lista e usar a lista em
http://www.mat.puc-rio.br/~nicolau/olimp/obm-l.html
O administrador desta lista é [EMAIL PROTECTED]
=


Re: [obm-l] Negocio da China !

2003-02-19 Por tôpico Eduardo Wagner
Ha um pequeno engano no problema 2.
Na verdade, deve-se provar que 
OY e' perpendicular a XY.

Abracos,
Wagner.


--
From: Paulo Santa Rita [EMAIL PROTECTED]
To: [EMAIL PROTECTED]
Subject: [obm-l] Negocio da China !
Date: Wed, Feb 19, 2003, 12:09 PM


 Ola Pessoal,

 Seguem abaixo as traduçoes ( do ingles ) de tres problemas de Olimpiadas da
 China.

 (1 - CHINA 1990 ) S e o conjunto de todos os sub-conjuntos de um dado
 conjunto X que teem um mesmo numero de elementos e F e uma funcao real
 definida sobre S tal que F(A)  1990 para algum elemento A de S. Sabe-se
 tambem que : F(B uniao C)=F(B)+ F(C)-1990 para todos elementos B e C de
 S que sejam disjuntos. Mostre que nos podemos encontrar um
 sub-conjunto Y de X tal que :

 F(D)   1990 para todo D contido em Y
 F(D) = 1990 para todo D contido em X-Y

 (2 - CHINA 1992 ) As diagonais de um quadrilatero ciclico (inscritivel)
 encontram-se em X. O circulo circunscrito ao triangulo ABX encontra o
 circulo circunscrito ao triangulo CDX em X e Y. Se O e o centro do circulo
 circunscrito ao quadrilatero ABCD e O,X e Y sao distintos dois a dois,
 mostre que OY e perpendicular a OX.

 (3 - CHINA 1994 ) Seja p(z)= z^N + An-1*z^(N-1) + ... + A0 um polinomio com
 coeficientes complexos. Mostre que nos podemos encontrar um ponto (numero
 complexo) z com modulo(z') = 1 e tal que
 modulo(p(z')) = 1 + modulo(A0).

 Um Abraco a Todos
 Paulo Santa Rita
 4,1209,190203






 _
 MSN Messenger: converse com os seus amigos online.
 http://messenger.msn.com.br

 =
 Instruções para entrar na lista, sair da lista e usar a lista em
 http://www.mat.puc-rio.br/~nicolau/olimp/obm-l.html
 O administrador desta lista é [EMAIL PROTECTED]
 =
=
Instruções para entrar na lista, sair da lista e usar a lista em
http://www.mat.puc-rio.br/~nicolau/olimp/obm-l.html
O administrador desta lista é [EMAIL PROTECTED]
=



[obm-l] Re: [obm-l] Dúvidas básicas...

2003-02-16 Por tôpico Wagner
Oi para todos !

x^0 indica a identidade do grupo C* para a operação multiplicação.
Logo a.x^0 = a , para todo a pertencente a C* . Logo x^0 = 1

André T.



- Original Message -
From: Henrique Branco [EMAIL PROTECTED]
To: [EMAIL PROTECTED]
Sent: Thursday, February 13, 2003 8:42 PM
Subject: [obm-l] Dúvidas básicas...


 Pessoal,
 Tenho duas dúvidas que são bem básicas...
 Existe alguma demonstração (formal, de preferencia) sobre x^0 = 1 e 0! =
1?
 Sendo 0! o fatorial de zero.
 Grato,
 Henrique.

 =
 Instruções para entrar na lista, sair da lista e usar a lista em
 http://www.mat.puc-rio.br/~nicolau/olimp/obm-l.html
 O administrador desta lista é [EMAIL PROTECTED]
 =


=
Instruções para entrar na lista, sair da lista e usar a lista em
http://www.mat.puc-rio.br/~nicolau/olimp/obm-l.html
O administrador desta lista é [EMAIL PROTECTED]
=



[obm-l] Re: [obm-l] quetão1

2003-02-16 Por tôpico Wagner
Oi para todos!

Seja v a velocidade da correnteza e V a velocidade do barco.
V + v = 16/1 = 16
V - v = 16/2 = 8 = 2V = 24 = V = 12 km/h
12 + v = 16 = v = 4 km/h

André T.


- Original Message -
From: elton francisco ferreira [EMAIL PROTECTED]
To: [EMAIL PROTECTED]
Sent: Sunday, February 16, 2003 3:14 PM
Subject: [obm-l] quetão1


 Um barco percorre 16km em 1 h, navegando a favor da
 corrente; para retornar pelo mesmo trajeto, demora 2
 h. Qual é a velocidade do barco e a da corrente?

 ___
 Busca Yahoo!
 O serviço de busca mais completo da Internet. O que você pensar o Yahoo!
encontra.
 http://br.busca.yahoo.com/
 =
 Instruções para entrar na lista, sair da lista e usar a lista em
 http://www.mat.puc-rio.br/~nicolau/olimp/obm-l.html
 O administrador desta lista é [EMAIL PROTECTED]
 =


=
Instruções para entrar na lista, sair da lista e usar a lista em
http://www.mat.puc-rio.br/~nicolau/olimp/obm-l.html
O administrador desta lista é [EMAIL PROTECTED]
=



Re: [obm-l] Mais probabilidade e combinatoria

2003-02-13 Por tôpico Wagner
Oi para todos !

2)Vamos achar em quantas posições diferentes o cubo pode se encontrar:
-Primeiro vamos enumerar as faces de 1 a 6.
-Com a face 1 voltada para baixo temos 4 posições que são rotações de
90º do cubo.
-Analogamente temos 6.4 = 24 posições diferentes para o cubo
O nº de possibilidades de pintura para o cubo, quando a posição dele
importa é 6! = 720
O nº de formas que o cubo pode ser pintado é 720/24 = 30 possibilidades

3)Não seria somatório duplo ao invés de produto de somatórios
(veja se existe um sinal de multiplicação entre eles) ?

André T.


- Original Message -
From: amurpe [EMAIL PROTECTED]
To: [EMAIL PROTECTED]
Sent: Thursday, February 13, 2003 8:25 AM
Subject: [obm-l] Mais probabilidade e combinatoria


 Por favor me ajudem na resolução desses problemas.

 1) O mes de outubro tem 31 dias .Numa certa cidade chove
 5 dias , no mes de outubro.Qual a probabilidade de não
 chover no primeiro e segundo dia de outubro?
 resp: 65/93.

 Obs: imaginei que fosse 1-5/31=26/31.

 2) ITA-71; dispomos de 6 cores diferentes.Cada face de
 um cubo será pintada com uma cor diferente, de forma que
 as 6 cores sejam utilizadas.De quantas maneiras
 diferentes isto pode ser feito , se uma maneira é
 considerada identica a outra , desde que possa ser
 obtida a partir desta por rotação do cubo?
 resp: 30

 3)ITA-68.
 Sejam a1, a2,...an numeros reais.A expressão (
 a1+a2+.an)^2 é igual a ...

 resp: Somatorio de i variando de 1 a n multiplicado por
 somatorio de j variando de 1 a n de ai.aj.



 desde já muito obrigado pela ajuda.

 um abraço.

 Amurpe


 __
 E-mail Premium BOL
 Antivírus, anti-spam e até 100 MB de espaço. Assine já!
 http://email.bol.com.br/


 =
 Instruções para entrar na lista, sair da lista e usar a lista em
 http://www.mat.puc-rio.br/~nicolau/olimp/obm-l.html
 O administrador desta lista é [EMAIL PROTECTED]
 =



=
Instruções para entrar na lista, sair da lista e usar a lista em
http://www.mat.puc-rio.br/~nicolau/olimp/obm-l.html
O administrador desta lista é [EMAIL PROTECTED]
=



[obm-l] função zeta

2003-02-12 Por tôpico Wagner



Oi para todos!

 Porquê zeta( 0 ) = -1/2 
?
 
infinito
 Se zeta( x ) = 
SOMATÓRIO 1/(n^x) , zeta( 0 ) não deveria ser igual a infinito 
?
n 
= 1

André T.


[obm-l] cálculo

2003-02-09 Por tôpico Wagner



Oi pessoal !

 Alguém conhece uma demonstração 
usando cálculo para a fórmula do volume de uma pirâmide?

André T.


Re: [obm-l] Olá- Construção Geométrica

2003-02-06 Por tôpico Eduardo Wagner
Caro Edu:

Este eh o problema 89 do livro da Olimpiada Brasileira
de Matematica, 1a a 8a.
Adquira este livro. Voce vai ver a solucao detalhada
desta problema e vai conhecer muitos outros problemas
interessantes. Entre em contato com a secretaria da
OBM pelo e-mail [EMAIL PROTECTED] ou pelo telefone
25295077.


--
From: Eduardo [EMAIL PROTECTED]
To: [EMAIL PROTECTED]
Subject: [obm-l] Olá- Construção Geométrica
Date: Thu, Feb 6, 2003, 9:29 PM


 Olá, pessoas

 Acabo de ingressar na lista e gostaria de propor um problema:

 Dados dois segmentos a e b, construir um segmento de medida (a^4+b^4)^1/4

 Se alguém puder me ajudar...

 abraços

 Edu
 ---
 Outgoing mail is certified Virus Free.
 Checked by AVG anti-virus system (http://www.grisoft.com).
 Version: 6.0.449 / Virus Database: 251 - Release Date: 27/1/2003

 ___
 Busca Yahoo!
 O serviço de busca mais completo da Internet. O que você pensar o Yahoo!
encontra.
 http://br.busca.yahoo.com/

 =
 Instruções para entrar na lista, sair da lista e usar a lista em
 http://www.mat.puc-rio.br/~nicolau/olimp/obm-l.html
 O administrador desta lista é [EMAIL PROTECTED]
 =
=
Instruções para entrar na lista, sair da lista e usar a lista em
http://www.mat.puc-rio.br/~nicolau/olimp/obm-l.html
O administrador desta lista é [EMAIL PROTECTED]
=



Re: [obm-l] O armario e o corredor

2003-02-03 Por tôpico Wagner
Oi para todos!

Mas ai você está assumindo que o eixo de rotação é conhecido, o que não é
dito
no enunciado.

André T.

- Original Message -
From: Cláudio (Prática) [EMAIL PROTECTED]
To: [EMAIL PROTECTED]
Sent: Friday, January 31, 2003 4:13 PM
Subject: Re: [obm-l] O armario e o corredor


 Caros Salvador e Paulo:

 Tenho a impressão de que o retângulo de maior área que efetivamente faz a
 curva sofrendo uma rotação de 90 graus é um quadrado de lado 1/raiz(2)
(área
 = 1/2). Se não for necessário que o retêngulo sofra uma rotação, então o
 quadrado de lado 1 é um candidato melhor - ele apenas muda a direção de
seu
 deslocamento.

 No entanto, o candidato mais forte que eu consegui imaginar é um
semicírculo
 de raio 1 (área Pi/2).

 Claudio.

 - Original Message -
 From: Salvador Addas Zanata [EMAIL PROTECTED]
 To: [EMAIL PROTECTED]
 Sent: Friday, January 31, 2003 12:03 PM
 Subject: Re: [obm-l] O armario e o corredor



 Oi Paulo,


 Encontrei esse problema num livrinho chamado Unsolved Problems in
 Geometry, ou coisa parecida. Eh da editora Springer. O livro e bem legal,
 tem um colecao enorme de problemas intuitivos, todos MUITO dificeis.

 Faz bastante tempo que li, mas pelo que me lembro, o Conway provou que
 esse maximo existe, mas o valor exato nao e conhecido. Ele deu tambem
 estimativas e sugeriu formas para
 este objeto (formas parecidas com alteres, coisa razoavelmente natural).

 Imagino que os metodos sejam variacionais, mas nao vi nada sobre esse
 problema. Se voce morar em Sao Paulo, na biblioteca do IMEUSP, voce
 encontrara esse livro. Em cada problema, sao citadas referencias com
 resultados parciais.


 Boa sorte,

 Salvador



 On Fri, 31 Jan 2003, Paulo Santa Rita wrote:

  Hi Salvador e demais
  colegas desta lista ... OBM-L,
 
  Gostei do problema. Voce pode falar mais um pouco sobre ele ? Se eu
  resolve-lo ou conseguir algum progresso significativo mostro ao Conway e
  publico aqui nesta lista.
 
  Desde agradeco.
 
  Um abraco
  Paulo Santa Rita
  6,1043,310103
 
  From: Salvador Addas Zanata [EMAIL PROTECTED]
  Reply-To: [EMAIL PROTECTED]
  To: [EMAIL PROTECTED]
  Subject: Re: [obm-l] O armario e o corredor
  Date: Thu, 30 Jan 2003 22:45:50 -0200 (EDT)
  
  
  
  Caros amigos,
  
  Um problema pelo que eu sei, em aberto, relacionado a esse consiste no
  seguinte:
  
  Dado um corredor com 1 metro de largura, que faz uma curva de 90
graus
 e
  continua com a mesma largura, qual e a maior area possivel que pode
fazer
  essa curva? Observe que o formato dessa area pode ser qualquer, e
  obviamente ela e suposta rigida. E claro que o maior segmento que essa
  area contem e limitado, mas isso nao ajuda muito.
  
  O John Conway fez algumas coisas parciais sobre isso.
  
  
  Abraco,
  
  
  Salvador
  
  
  
  On Thu, 30 Jan 2003, Paulo Santa Rita wrote:
  
Ola Claudio e demais
colegas desta lista ... OBM-L,
   
Resposta correta ! Com sinceridade alertei que o problema, nao
 obstante
simples, tinha uma solucao surpreendente !
   
Em verdade esse problema me foi sugerido em uma mudanca la em casa,
  quando
eu ainda era menino : meu pai e tios tentavam arrastar um grande
 armario
atraves de um corredor em forma de L, quando entao os sucessivos
  fracassos
os levaram a suspeitar que era impossivel, sem saberem justificar.
   
Provando ( Garantindo ! Ele nao conhecem Calculo. ) que era
 impossivel,
  eu
os convenci a desmontarem o armario, previamente. So depois de
muitos
  anos
vim a saber que havia um problema de Calculo Diferencial muito
 parecido.
   
Eu nao acompanhei todos os calculos que voce efetuou, mas a ideia
  contida no
fragmento abaixo esta correta e e o insight que mata a questao. Se
eventualmente houver algum erro no algebrismos ( na burocracia ) e
sem
duvida apenas uma desatencao.
   
Vou propor agora um problema que nao e facil. Para que ele possa ser
digerido, vou coloca-lo na forma de sub-problemas :
   
PROBLEMA : Seja Q um quadrado de lado unitario. Mostre que, qualquer
 que
seja a forma como colocarmos no interio de Q dois outros quadrados
de
  lados
L1 e L2, se L1 + L2  1 entao estes dois outros quadrados terao ao
 menos
  um
ponto em comum.
   
Esse e um dos problemas do Paul Erdos. Ja foi proposto aqui nesta
 lista.
A ideia e encontrar uma demonstracao rigorosa, analitica, que nao
 lance
  mao
de intuicoes geometricas contestaveis.
   
SUGESTAO : Podemos representar Q como a regiao do R^2 na qual as
  coordenas
(X,Y) de qualquer ponto obedece as condicoes :
   
0 = X = 1
0 = Y = 1
   
Precisamos encontrar uma maneira de garantir que os quadrados de
lados
  L1 e
L2 estejam confinados em Q. Convencionemos, pois, que :
   
1) O quadrado de lado L1 (L2) tem vertices ABCD (EFGH) com o lado AD
  (EH)
inclinado de ALF (BET) em relacao aos eixo das abscissas.
2) A (E) e o vertice de menor ordenada. Se dois vertices tiverem
a
  

Re: [obm-l] Um probleminha, nada mais.

2003-02-03 Por tôpico Wagner
Olá para todos!

Suponha que todos os túneis tenham a mesma chance de serem escolhidos,
1/3 dos prisioneiros chega a liberdade em 1 hora.
1/3 dos prisioneiros chega a liberdade em 3 horas.
1/3 dos prisioneiros voltam a situação inicial.
Supondo que eles não voltem a escolher o túnel que não os levou a liberdade:
1/6 dos prisioneiros chega a liberdade em 7 horas.
1/6 dos prisioneiros chega a liberdade em 9 horas.
MÉDIA = (1/3).1 + (1/3).3 + (1/6).7 + (1/6).9 = (2 + 6 + 7 + 9)/6 = 24/6 =
MÉDIA = 4 horas.
Outro problema interessante é se os túneis forem indistinguíveis.

André T.

- Original Message -
From: cfgauss77 [EMAIL PROTECTED]
To: Lista OBM [EMAIL PROTECTED]
Sent: Monday, February 03, 2003 12:18 AM
Subject: [obm-l] Um probleminha, nada mais.


 Vou propor um probleminha que eu achei interessante.

  Em uma cela, há passagem secreta que conduz a um porão
 de onde partem trên túneis. O primeiro túneo dá acesso à
 liberdade em 1 hora; o segundo, em 3 horas; e o terceiro
 leva ao ponto de partida em 6 horas. Em média, os
 prisioneiros que descobrem os túneis conseguem escapar
 da prisão em quanto tempo?

Não é difícil mas é inteligente.




 __
 E-mail Premium BOL
 Antivírus, anti-spam e até 100 MB de espaço. Assine já!
 http://email.bol.com.br/


 =
 Instruções para entrar na lista, sair da lista e usar a lista em
 http://www.mat.puc-rio.br/~nicolau/olimp/obm-l.html
 O administrador desta lista é [EMAIL PROTECTED]
 =


=
Instruções para entrar na lista, sair da lista e usar a lista em
http://www.mat.puc-rio.br/~nicolau/olimp/obm-l.html
O administrador desta lista é [EMAIL PROTECTED]
=



Re: [obm-l] Probabilidade

2003-02-02 Por tôpico Wagner



Oi para todos !

Exatamente, esse é o problema que tenho em mente. A 
minha dúvida é que dependendo de como o problema é encarado, um mesmo 
evento
pode ter probabilidades diferentes. Eu pensei nesse 
problema usando um pacote como os de "skittles". Suponha que você abra o pacote 
enão 
olhe dentro do pacote. Qual a probabilidade de que 
uma bala retirada aleatoriamente seja amarela? Se você fizer a contagem, verá 
que essa
probabilidade é igual a porcentagem em que essa bala é fabricada, ou seja 20%. Mas suponha que 
você olhe dentro do pacote, tire todas as 
balas, conte-as e coloque elas de novo no pacote. Qual a probabilidade de que uma bala retirada 
aleatoriamente seja amarela? Nesse caso a 
probabilidade depende da distribuição das 
balas.
A minha pergunta é: Existe um modo mais apropriado 
de encarar essa questão, ou o fato de saber qual é a distribuição implica que os 
dois
eventos são diferentes?
Acho que é a segunda hipótese, por que a partir do 
momento que se conhece a distribuição, o pacote deixa de ser um pacote 
qualquer.
Mas não estou muito certo, se alguém puder 
confirmar ficarei grato.

André T.


  - Original Message - 
  From: 
  Cláudio (Prática) 
  To: [EMAIL PROTECTED] 
  Sent: Friday, January 31, 2003 12:34 
  PM
  Subject: Re: [obm-l] Probabilidade
  
  Neste caso, eu diria que a resposta à sua 
  pergunta é sim.
  
  Por exemplo, se não houver bala nenhuma de anis, 
  então P(primeira bala retirada é de anis) = 0. No entanto, se houver pelo 
  menos 1 bala de anis, P(anis)  0.
  
  Não sei se isso é o que você tinha em mente. 
  
  
  Talvez uma formulação mais precisa seja a 
  seguinte:
  
  Você tem um gerador de números aleatórios que, a 
  cada segundo, gera um dos números 1, 2, 3, 4 ou 5 (os cinco sabores), cada um 
  com probabilidade = 20% (todos os sabores produzidos na mesma quantidade), e 
  imprime este número numa fita que se desloca lateralmente com velocidade 
  constante. Assim, a fita contém uma série de números, um do lado do outro 
  (espero que a descrição seja inteligível).
  
  Você também tem um leitor de números, que lê 
  números desta fita a intervalos de tempo aleatórios (ou seja, o número lido 
  corresponde à primeira bala retirada do saco e os números não lidos 
  subsequentes às demais balas deste saco - acho que isso modela o fato de que o 
  número de balas em cada pacote é aleatório). 
  A distribuição dos intervalos de tempo T entre 
  duas leituras é dada por:
  P(T=1) = p1, P(T=2) = p2, ..., onde, para cada i, 0 = pi = 1 e 
  p1 + p2 +  = 1.
  ( por exemplo, podemos ter P(T=k) = 1/2^k, ou 
  então, 6/(Pi^2*k^2) ).
  
  Neste caso, eu pergunto:
  Um número é lido. Qual a probabilidade dele 
  sero "1"? Essa probabilidadedepende da distribuiçao de 
  T?
  
  Um abraço,
  Claudio.
  
- Original Message ----- 
    From: 
Wagner 
To: [EMAIL PROTECTED] 
Sent: Thursday, January 30, 2003 8:42 
PM
Subject: Re: [obm-l] 
Probabilidade

Oi para todos!

A "distribuição" a que me referi é o número 
total de balas e a quantidade de balas de cada sabor 
independente da organização geométrica em que 
as balas se encontram dentro do pacote, uma
vez que o evento tirar uma bala é aleatório 
eessa tal organização também seria aleatória, essa
organização passa a ser 
irrelevante.
OBS: Só para lembrar, o evento de tirar uma 
bala do pacote é aleatório

André T.



  - Original Message - 
  From: 
  Cláudio (Prática) 
  To: [EMAIL PROTECTED] 
  Sent: Thursday, January 30, 2003 6:00 
  PM
  Subject: Re: [obm-l] 
  Probabilidade
  
  O que você quer dizer com distribuição de 
  balas dentro de um pacote - é simplesmente o número de balas de cada sabor 
  ou envolve algum tipo de arranjo geométrico ou ordenaçào das balas (como 
  num pacote de Halls, por exemplo)?
  
  
- Original Message - 
From: 
Wagner 

To: [EMAIL PROTECTED] 
Sent: Wednesday, January 29, 2003 
2:24 PM
Subject: [obm-l] 
Probabilidade

Oi pessoal !

Deêm uma olhada nessa minha 
dúvida:

Uma fábrica de balas produz balas de 5 
sabores diferentes (todas as balas são produzidas na mesma 
quantidade)
Essas balas são embaladas aleatoriamente, 
de modo que os pacotes podem conter qualquer número de balas 
(pelo
menos uma).
A probabilidade de abrir um pacote qualquer 
e a primeira bala tirada dele ser de um determinado sabor depende da 
distribuição 
das balas dentro do pacote?

André 
  T.


[obm-l] Re: [obm-l] Re: [obm-l] Domínó

2003-01-30 Por tôpico Wagner



Oi para todos !

Posso estar enganado mas essa pergunta é 
praticamente impossível de ser respondida sem usar teoria dos jogos

André T.




  - Original Message - 
  From: 
  Eduardo Azevedo 
  To: [EMAIL PROTECTED] 
  Sent: Thursday, January 30, 2003 1:52 
  AM
  Subject: [obm-l] Re: [obm-l] Domínó
  
  No domino os jogadores querem ganhar, e nao jogam 
  aleatoriamente. Pra responder essa pergunta (que deve ser dificil) voce vai 
  ter que definir qual vai ser o comportamento dos jogadores.
  
  
  
- Original Message - 
From: 
Tertuliano 
Carneiro 
To: [EMAIL PROTECTED] 
Sent: Wednesday, January 29, 2003 5:18 
PM
Subject: [obm-l] Domínó

Há algum tempo, um colega meu me propôs um problema quando estávamos 
jogando uma partida de dominó em duplas. Ele me perguntou qualé a 
probabilidade deum dos quatro jogadores levar um "chico romero". Eu 
explico: aqui na Bahia, nós chamamos de "chico romero" ao inusitado fato de 
um dos jogadores nao conseguir "colar" nenhuma das peças, ou seja, quando 
ele termina o jogo com as sete peças na mão.
No momento eu ate consegui rabiscar alguma coisa, mas não fui muito 
longe. Convido vcs a pensarem no problema,pois me pareceu bastante 
interessante, embora nao tenha conseguido concluir muita coisa.

Fui!
Tertuliano Carneiro.


Yahoo! GeoCitiesTudo 
para criar o seu site: ferramentas fáceis de usar, espaço de sobra e 
acessórios.


[obm-l] Re: [obm-l] Re: [obm-l] Dúvida???

2003-01-27 Por tôpico Wagner



Oi para todos !

Desculpe o descuido, faltou dizer que x deve ser 
primo. Para n  1 oconjunto tem pelo menos um número primo.
Mas me ocorreu uma dúvida, a afirmação vale para 
n=1?

André T.



  - Original Message - 
  From: 
  A. C. 
  Morgado 
  To: [EMAIL PROTECTED] 
  Sent: Sunday, January 19, 2003 1:04 
  PM
  Subject: Re: [obm-l] Re: [obm-l] 
  Dúvida???
  Por exemplo, se n=5, o conjunto eh {1, 2, ...,10}. Considere o 
  subconjunto {1, 2, 3, 4, 5, 6}. Se x=4, um elemento do subconjunto que não eh 
  multiplo de x eh o 6. Logo, de acordo com a prova dada 4 e 6 sao primos entre 
  si!MorgadoWagner wrote:
  



Oi para todos !

Sim. Aqui vai a prova :
Suponha que m elementos do subconjunto sejam 
múltiplos de x.
Para x  1, temos m  n + 1 . Logo existe 
pelo menos um elemento do subconjunto que não
é múltiplo de x . Seja y esseelemento . 
Como x = 2n , então pelo menos x e y
são primos entre si.

André T.



  - 
  Original Message - 
  From: 
  Danilo Artigas 
  To: obm-l 
  
  Sent: 
  Sunday, January 19, 2003 2:24 AM
  Subject: 
  [obm-l] Dúvida???
  
   Por que podemos garantir que em 
  qualquer subconjunto com n + 1 elementos do conjunto {1, 2, 3,..., 
  2n} existem pelo menos doiselementos que são primos entre 
  si?


[obm-l] Sequências

2003-01-27 Por tôpico Wagner



Provar que todo cubo de um número inteiro é a 
diferença de dois quadrados de números inteiros

André T.


[obm-l] Re: [obm-l] Dúvida???

2003-01-19 Por tôpico Wagner



Oi para todos !

Sim. Aqui vai a prova :
Suponha que m elementos do subconjunto sejam 
múltiplos de x.
Para x  1, temos m  n + 1 . Logo existe 
pelo menos um elemento do subconjunto que não
é múltiplo de x . Seja y esseelemento . Como 
x = 2n , então pelo menos x e y
são primos entre si.

André T.



  - Original Message - 
  From: 
  Danilo Artigas 
  To: obm-l 
  Sent: Sunday, January 19, 2003 2:24 
  AM
  Subject: [obm-l] Dúvida???
  
   Por que podemos garantir que em qualquer 
  subconjunto com n + 1 elementos do conjunto {1, 2, 3,..., 2n} existem 
  pelo menos doiselementos que são primos entre 
si?


[obm-l] Re: [obm-l] sistema de equações

2003-01-19 Por tôpico Wagner



Oi pessoal !

d = 8D + 24
D + d + 24 = 344 =

d - 8D = 24
d + D = 320

André T.




  - Original Message - 
  From: 
  [EMAIL PROTECTED] 
  
  To: [EMAIL PROTECTED] 
  Sent: Sunday, January 19, 2003 3:54 
  AM
  Subject: [obm-l] sistema de 
equações
  Olá pessoal, Estou com dúvidas nesta questão da 
  FVG: (F.G.V-SP)Numa divisão, o quociente é 8 e o resto, 24. Sabe-se 
  que a soma do dividendo, do divisor e do resto é 344. Então, a diferença 
  dividendo menos divisor é: Resp: 248 Obs: Tentei aplicar a 
  divisão euclidiana (d=D*q + r) mas não consegui montar um sistema de equações, 
  pois como os exercícios do meu fascículo estão separados por tópicos, sei que 
  esta questão se resolve por sistema de equações, mas o problema está em 
  montá-la. 


Re: [obm-l] Livro Geometria

2003-01-16 Por tôpico Eduardo Wagner
Title: Re: [obm-l] Livro Geometria



Geometria II foi um livro que escrevemos em 1974 (quando muito jovens)
e editado pela Livraria Francisco Alves. Ficou bom, esgotou-se rapidamente
e a editora faliu. Durante as duas ultimas decadas esse livro foi intensamente
pirateado, xerocado e vendido por oportunistas, bandidos, biltres, trapaceiros,
ladroes e toda a classe de gente que se aproveita do trabalho alheio para 
ganhar algum dinheiro.
Resolvemos entao no fim do ano passado, fazer uma nova impressao do
livro original, que deve aparecer nas livrarias no proximo mes.
O livro Geometria II trata de geometria metrica. Respondendo a sua
pergunta (que eh perfeitamente logica), existiu sim o Geometria I, que
tratava de geometria de posicao. Este esta realmente desaparecido.
Na verdade, nao era tao bom (eu acho).

Abracos,

E. Wagner. 

--
From: Leonardo Borges Avelino [EMAIL PROTECTED]
To: [EMAIL PROTECTED]
Subject: [obm-l] Livro Geometria
Date: Thu, Jan 16, 2003, 10:07 PM


Caros amigos:

Estava na casa de meu amigo e ele me mostrou um livro impressionante, que se chama: Geometria II dos prof.s Eduardo Wagner, Augusto Morgado e Miguel.
Pergunta:

Onde consigo este livro? e (desculpem-me se a pergunta for idiota) se existe o Geometria I?


Valeu!!
Leonardo Borges






Re: [obm-l] Re: [obm-l] Triângulos Isósceles e Bissetrizes

2003-01-13 Por tôpico Eduardo Wagner
Title: Re: [obm-l] Re: [obm-l] Triângulos Isósceles e Bissetrizes





--
From: Cláudio \(Prática\) [EMAIL PROTECTED]
To: [EMAIL PROTECTED]
Subject: [obm-l] Re: [obm-l] Triângulos Isósceles e Bissetrizes
Date: Thu, Jan 9, 2003, 11:42 AM


Caro Eduardo:

Obviamente, esta é a solução que vai para o LIVRO.

No entanto, pelo menos para mim, a maior dificuldade que existe em problemas de geometria é determinar a construção auxiliar (no caso, o segmento EF e, por conseguinte, paralelogramo BDFE) que mata o problema.

Existe alguma maneira sistemática de se buscar estas construções auxiliares ou infelizmente, só podemos contar com a experiência e a esperança de algum insight genial? E se, por acaso, existir tal maneira, você recomenda alguma bibliografia em particular?

A resposta eh nao. Se existisse, a atividade de resolver problemas nao teria
a menor graca. Mas as tentativas em obter construcoes auxiliares nao ocorrem inteiramente ao acaso. Tracar uma paralela, uma perpendicular, fazer uma rotacao,
uma simetria (entre outras coisas), frequentemente permitem reunir os dados
do problema em outra posicao, permitindo encontrar uma relacao entre eles.
Observe na resolucao deste problema, qual foi a ideia da criacao do paralelogramo:
conectar as bissetrizes iguais formando um triangulo isosceles! Isto eh algum
metodo. Se em algum problema ha dois segmentos iguais, devemos imaginar
uma maneira de conecta-los.
A melhor fonte para conseguir construcoes auxiliares eh certamente a experiencia.
Conhecer muitos problemas e observar cuidadosamente o porque da construcao.

Eu pergunto isso porque tenho observado que muitos problemas (possivelmente todos) que são resolvidos via estas construções auxiliares podem também ser resolvidos via trigonometria, apesar destas soluções serem muito mais longas e deselegantes, envolvendo uma quantidade razoável de álgebra. Minha suspeita é que talvez haja alguma relação profunda e geral entre soluções via construção auxiliar e soluções trigonométricas.

Sua suspeita nao eh so sua. Muitas vezes se consegue obter a solucao via 
construcoes auxiliares depois da solucao trigonometrica. Mas, nem sempre.

Um abraço,
Claudio Buffara. 
- Original Message - 
From: Eduardo Wagner mailto:[EMAIL PROTECTED] 
To: [EMAIL PROTECTED] mailto:[EMAIL PROTECTED] 
Sent: Saturday, January 11, 2003 12:27 AM
Subject: Re: [obm-l] Triângulos Isósceles e Bissetrizes



O problema é: Prove que se um triângulo tem duas bissetrizes internas iguais, então ele é isósceles.


Solucao:

Desenhe o triangulo ABC e as bissetrizes BD e CE.
Construa o paralelogramo BDFE e trace CF.

Assinale os angulos:
ABC = 2b, ACB = 2c, EFD = b, DFC = x, DCF = y.

EF = BD = EC. Logo, b + x = c + y.

Suponha que os angulos B e C sejam desiguais,
B  C, por exemplo, e observe as implicacoes:

B  C
b  c
x  y
DC  DF
DC  BE
DBC = b  c = EBC (contradicao).

Logo, os angulos B e C sao iguais.

Abracos,

E. Wagner.


















Re: [obm-l] Diversos

2003-01-13 Por tôpico Eduardo Wagner
Title: Re: [obm-l] Diversos





--
From: Leonardo Borges Avelino [EMAIL PROTECTED]
To: [EMAIL PROTECTED]
Subject: [obm-l] Diversos
Date: Sun, Jan 12, 2003, 10:15 PM




Caros amigos da lista:

1)Estava um dia em minha casa e comecei a estudar construções geométricas( com régua lisa e compasso, somente) e bolei alguns exercícios para eu mesmo resolver. Por exemplo: construir duas retas perpendiculares, logo após contruir um ângulo de 30º, logo após, contruir um ângulo de 15º, logo após contruir um triângulo retângulo de ângulos agudos 30 e 60 graus, isso como incentivo para continuação no estudo de construções que auxiliam na geometria. A questão é: Se não me engano, vi um livro que se chama Contruções Geométricas do prof. Eduardo Wagner, da Coleção do professor de Matemática. Este livro é completo para o estudo de questões sobre este tema da OBM?
Sem duvida.
Obs: Estas questões para vocês são moleza, mas só citei pois comecei a estudar tal assunto somente agora.

2) Caro Faelccmm, vc estava perguntando sobre exercícios de geometria, né? Então gostaria de citar uns livrinhos que ajudam para começar: Geometria do Oswaldo Marcondes e Ed. do Brasil de 1969; Geometria plana Vol9 da coleção do Iezzi.
Gostaria de dizer também, que eu acho legal se vc pegasse algumas provas de matemática do Colégio Naval, pois tem exercícios de geometria que eu considero interessantes.

Obs: Citei estes livros pois tratam de assuntos importantes e demonstram alguns teoremas, mas não são base total para a OBM.






Re: [obm-l] matrizes

2003-01-11 Por tôpico Wagner



Oi para todos! 


Desculpe a distração na última mensagem. Toda 
matriz [(a11 = x) e (a21 = 2x/3)] satizfaz X.

André T.




  - Original Message - 
  From: 
  [EMAIL PROTECTED] 
  
  To: [EMAIL PROTECTED] 
  Sent: Saturday, January 11, 2003 4:57 
  PM
  Subject: [obm-l] matrizes
  olá pessoal, 
  Gabriel Haeser, vou reafirmar novamente que enviei a questão como 
  está no meu caderno de exercícios com a resalva de ser A*X=3X ao invés de 3x 
  (esta segunda é como está no enunciado). Quanto a unicidade da solução a 
  questão dá 5 alternativas (todas matrizes coluna) e entre elas a matriz 
  [(a11=3) e (a21=2)] que alguém da lista enviou, mas não me lembro quem. Parece 
  que foi o Bruno. Gostaria que quem enviou, comentasse aos outros colegas aqui 
  da lista para sabermos se sua resposta está realmente correta. Pelo menos, 
  bate perfeitamente com o gabarito.  



  1   2   3   >